Download as pdf or txt
Download as pdf or txt
You are on page 1of 55

VISIONIAS

www.visionias.in
ANSWERS & EXPLANATION
GENERAL STUIDES (P) 2023 – ABHYAAS TEST – 4039

Q 1.C
• A payment aggregator, also known as a merchant aggregator, is a third-party payment solutions provider
that offers merchant onboarding services. With a payment aggregator, merchants can accept multiple
modes of payment. Some examples of payment aggregators include Citrus, Billdesk, Instamojo,
CCAvenue and PayUMoney etc.
• A payment gateway is a payment processing software. Payment gateways only deal with online
transactions and are equipped with restricted payment options. Examples of popular payment
gateways include CCAvenue Payment Gateway, PayUBiz India Payment Gateway, Direcpay Payment
Gateway, and Razorpay Payment Gateway etc. Also in India, most banks act as payment gateways.
• A payment aggregator and payment gateway might seem to do the same job. However, the fact is – there
are several differences between them.
o A payment aggregator allows merchants to offer various payment options to their customers. A
payment gateway provides the underlying technology for fund transfer. Hence statement 1 is
correct.
o The main difference between a payment aggregator and a payment gateway is that the former handles
funds while the latter provides technology.
o While a payment gateway is an intermediary, the payment aggregator is the interface where the
payment gateway processes the transactions. Most payment aggregators own payment gateways to
offer a variety of exclusive services to their merchant customers. Hence statement 2 is not correct.
o Payment gateways only deal with online transactions whereas aggregators deal with both online
and offline transactions. Hence statement 3 is correct.
o Private fintech companies can own payment aggregators, while a payment gateway can be owned by
banks (both private and public) and payment aggregators.
o Payment aggregators can offer a payment gateway but the reverse is not true. Payment aggregators in
India offer payment gateways to merchants and ask them for specific payment gateway charges.
Initially, payment gateways in India were primarily offered by leading public and private banks.
However, there are several other payment aggregators in India offering similar payment suites to
merchants all across the country.
o Payment gateway aggregators like Citrus, Billdesk, Instamojo, CCAvenue and PayUMoney also
offer online payment gateway services to different merchants.

Q 2.C
• Retail inflation in India is measured by the consumer price index (CPI). It is influenced by a number of
factors.
• A general fall in international commodity prices is likely to reduce the rate of inflation in India as it
will discourage the exports of commodities from India augmenting the domestic supply. At the same time,
it may lead to cheap imports of commodities in India. This will help to reduce domestic prices. Thus, a
fall in international commodity prices is unlikely to increase the rate of retail inflation in India.
Hence, option 1 is correct.
• Adverse weather conditions like unseasonal rains and excessive heat are likely to affect crop production in
India. This reduced production of agri commodities will lead to higher food prices. Thus it will increase
the rate of retail inflation in India. Hence, option 2 is not correct.
• Reduction in excise and customs duties would reduce the prices of commodities and hence unlikely to
increase the rate of retail inflation. Hence, option 3 is correct.

1 www.visionias.in ©Vision IAS


• Tight monetary policy is characterized by the hike in policy rates (Repo rate) by the Reserve Bank of
India. Such a policy is followed to reduce overall liquidity in the market. The increase in interest rates will
reduce the liquidity in the market and thus help to slow down the prices of commodities. Hence, it is
likely to reduce the rate of retail inflation. Hence, option 4 is correct.

Q 3.A
• About World Trade Organisation (WTO):
o The World Trade Organization is the only international organization that deals with the rules of trade
between countries.
o Founded in 1995, the WTO is run by its 164 members, and all decisions are taken through consensus
and any member can exercise a veto.
o It is the successor to the General Agreement on Tariffs and Trade (GATT), a group founded in 1948
whose rules created the modern multilateral trading system.
• The Trade-Related Investment Measures (TRIMs) Agreement applies only to measures that affect
trade in goods.
o It recognizes that certain measures can restrict and distort trade, and states that no member shall apply
any measure that discriminates against foreigners or foreign products (i.e. violates “national
treatment” principles in GATT).
o It also outlaws investment measures that lead to restrictions in quantities (violating another principle
in GATT).
o An illustrative list of TRIMs agreed to be inconsistent with these GATT articles is appended to the
agreement.
▪ The list includes measures that require particular levels of local procurement by an
enterprise (“local content requirements”). It also discourages measures that limit a company’s
imports or set targets for the company to export (“trade balancing requirements”). Hence option
(a) is the correct answer.
• The General Agreement on Trade in Services (GATS) is the first and only set of multilateral rules
governing international trade in services.
o Negotiated in the Uruguay Round, it was developed in response to the huge growth of the services
economy over the past 30 years.
o The General Agreement on Trade in Services has three elements:
▪ the main text containing general obligations and disciplines;
▪ annexes dealing with rules for specific sectors; and
▪ individual countries’ specific commitments to provide access to their markets, including
indications of where countries are temporarily not applying the “most-favored-nation” principle of
non-discrimination.
• The WTO’s Agreement on Trade-Related Aspects of Intellectual Property Rights (TRIPS),
negotiated in the 1986–94 Uruguay Round, introduced intellectual property rules into the multilateral
trading system for the first time.
o Creators can be given the right to prevent others from using their inventions, designs or other
creations — and to use that right to negotiate payment in return for others using them. These are
“intellectual property rights”.
▪ They take a number of forms. For example books, paintings and films come under copyright;
inventions can be patented; brandnames and product logos can be registered as trademarks; and so
on.
o Governments and parliaments have given creators these rights as an incentive to produce ideas that
will benefit society as a whole.
o The agreement covers five broad issues:
▪ how basic principles of the trading system and other international intellectual property agreements
should be applied
▪ how to give adequate protection to intellectual property rights
▪ how countries should enforce those rights adequately in their own territories • how to settle
disputes on intellectual property between members of the WTO
▪ special transitional arrangements during the period when the new system is being introduced.
• The Technical Barriers to Trade Agreement (TBT) tries to ensure that regulations, standards, testing
and certification procedures do not create unnecessary obstacles.

2 www.visionias.in ©Vision IAS


o It also recognizes countries’rights to adopt the standards they consider appropriate — for example, for
human, animal or plant life or health, for the protection of the environment or to meet other consumer
interests. Moreover, members are not prevented from taking measures necessary to ensure their
standards are met.
o The agreement also sets out a code of good practice for both governments and non- governmental or
industry bodies to prepare, adopt and apply voluntary standards.
o The agreement says the procedures used to decide whether a product conforms with relevant standards
have to be fair and equitable. It discourages any methods that would give domestically produced
goods an unfair advantage. The agreement also encourages countries to recognize each other’s
procedures for assessing whether a product conforms. Without recognition, products might have to be
tested twice, first by the exporting country and then by the importing country.

Q 4.C
• In order to enhance the safety, transparency and effective management of safe deposit lockers provided by
banks, apex banking regulator Reserve Bank of India (RBI) released a list of revised guidelines. They are
applicable to both new and existing safe deposit lockers and bank facilities for the safe custody of articles.
• In adherence with the same, banks are required to renew agreements with their existing locker users by
December 31, 2023. The guidelines followed observations made by the Supreme Court in Amitabha
Dasgupta vs United Bank of India. Some of the new locker rules include:
o While allotting lockers, banks have to enter into an agreement with the customer on duly stamped
paper, with a copy being provided to both parties. Locker agreements must clarify that the bank does
not maintain any record of locker contents and that it is not under any liability to insure the contents
of the locker against any risk whatsoever. Additionally, banks cannot offer insurance products to
customers for insuring locker contents. Hence statement 3 is correct.
o Banks will now be allowed to obtain a ‘term deposit’ at the time of allotment to a consumer,
covering three years’ rent and the charges for breaking open a locker if the hirer doesn’t
operate it or pay rent. Hence statement 1 is correct.
o Banks will be liable to pay in case of any loss of locker content resulting from the bank's negligence.
It has the responsibility to ensure that incidents like fire, theft/burglary/ robbery, dacoity, building
collapse do not occur in the bank's premises due to its own shortcomings, negligence and by any act
of omission/commission. Banks cannot claim that they bear no liability towards their customers for
loss of contents of the locker due to incidents mentioned above or attributable to fraud committed by
its employee(s). In this case, the banks' liability shall be for an amount equivalent to one
hundred times the prevailing annual rent of the safe deposit locker. Hence statement 2 is not
correct.
o The bank will not be liable for any damage or loss of locker contents caused by natural calamities or
acts of God such as earthquakes, floods, lightning, thunderstorms, or any act attributable to the
customer's sole fault or negligence.
o In the event of a merger, closure or shifting of a branch requiring physical relocation of lockers, banks
are required to give notices in two newspapers and inform customers at least two months in advance,
also offering the option to change or close the facility.
o Banks are required to put the entry, exit and premises of the locker-room under CCTV surveillance
and retain recordings for at least 180 days.
o Banks reserve the discretion to break open the locker if the rent remains pending for three years in a
row. They must, however, inform the user through a letter, email and SMS, and accord him
‘reasonable opportunity’ to withdraw the deposited contents.
• The Supreme Court in Amitabha Dasgupta vs United Bank of India stated that previously existing
regulations on locker management were “inadequate and muddled” with no uniformity in rules. It hence
urged the RBI to lay down comprehensive directions for locker facility and safe deposit management.

Q 5.D
• A liquidity trap is a situation in which monetary policy becomes ineffective at lowering interest rates,
because the nominal interest rate is already close to zero. Hence option 1 is not correct.
• Liquidity trap can lead to low or falling inflation, because a lack of demand for goods and services can
put downward pressure on prices. This can be caused by a deflationary spiral, in which falling prices
lead to a decrease in demand, which in turn leads to further price decreases. Hence option 2 is
correct.

3 www.visionias.in ©Vision IAS


• Consumption can be affected by a lack of demand for goods and services, which can lead to a decrease
in consumption. This can happen because people and businesses may prefer to hold cash. Hence option
3 is correct.
• In such a situation, people and businesses may prefer to hold cash rather than invest in securities, because
the opportunity cost of holding cash is so low. This can lead to a decrease in demand for goods and
services, and can make it difficult for the central bank to stimulate the economy through traditional
monetary policy. Hence option 4 is correct.

Q 6.A
• India’s agricultural exports are poised to scale a new peak in the financial year ending March 31, 2023.
Government data show the value of farm exports in April-December 2022, at $39 billion, was 7.9%
higher than the $36.2 bn for the corresponding period of the previous year. At the present rate, the record
$50.2 bn exports achieved in 2021-22 look set to be surpassed.
• However, equally significant are the imports of agri produce that, at $27.8 bn in Apr-Dec 2022, have
grown 15.4% over the $24.1 bn for Apr-Dec 2021. As a result, there has been a further shrinking of the
surplus on the farm trade account.
• The two big contributors to India’s agri-export growth have been rice and sugar.
o India in 2021-22 shipped out an all-time-high 21.21 million tonnes (mt) of rice valued at $9.66
billion. That included 17.26 mt of non-basmati (worth $6.12 billion) and 3.95 mt ($3.54 billion) of
basmati rice. In the current fiscal, the growth has been primarily led by basmati rice. Its exports have
gone up by 40.3% in value (from $2.38 billion in April-December 2021 to $3.34 billion in April-
December 2022) and 16.6% in quantity (2.74 mt to 3.20 mt) terms. The corresponding increases have
been less for non-basmati exports: 3.3% in value ($4.51 billion to $4.66 billion) and 4.6% in quantity
(12.60 mt to 13.17 mt).
o More spectacular perhaps is sugar. Sugar exports hit a record value of $4.60 billion in 2021-22, as
against $2.79 billion, $1.97 billion, $1.36 billion, and $810.90 million in the preceding four
fiscals. This fiscal has seen a further surge of 43.6%, from $2.78 billion in April-December 2021
to $3.99 billion in April-December 2022.
• More than a general export slowdown, it’s the growth in imports that should be cause for concern. This
has come mainly from three commodities.
o The first is vegetable oils, whose imports shot up from $11.09 bn in 2020-21 to $18.99 bn in
2021-22, and even more during the first nine months of 2022-23 over the same period of last
fiscal — from $14.04 bn to $16.10 bn or 14.7%. Imports now account for over 60% of the country’s
estimated 22.5-23 mt annual oil consumption.
o The second is cotton. India’s cotton exports reached an all-time-high of $4.33 bn back in 2011-12. It
remained at reasonably high levels until 2013-14 ($3.64 bn), before plunging to $1.62 bn by 2016-17
and $1.06 bn in 2019-20. There was a recovery thereafter to $1.90 bn in 2020-21 and $2.82 bn in
2021-22. But during this fiscal, not only have exports collapsed to $512.04 million in April-December
(from $1.97 billion in April-December 2021), imports have also soared from $414.59 million to $1.32
billion for the same period. In other words, India has turned from a net exporter to a net
importer of cotton.
o The third commodity is cashew. During April-December 2022, imports have posted a 64.6% rise to
$1.64 billion from $996.49 million in April-December 2021, even as exports of cashew products have
plummeted from $344.61 million to $259.71 million for the same period. A similar trend has been
witnessed in spices, with exports de-growing (from $2.95 billion to $2.75 billion) and imports edging
up ($955.75 million to $1.03 billion).
• Hence option (a) is the correct answer.

Q 7.C
• IMF loans are meant to help member countries tackle balance of- payments problems, stabilize their
economies, and restore sustainable economic growth. This crisis resolution role is at the core of IMF
lending. A key objective of recent lending reforms has therefore been to complement the traditional crisis
resolution role of the IMF with additional tools for crisis prevention.
• Unlike development banks, the IMF does not lend for specific projects. Instead, the IMF provides
financial support to countries hit by crises to create breathing room as they implement policies that restore
economic stability and growth. It also provides precautionary financing to help prevent crises. IMF
lending is continuously refined to meet countries’ changing needs.
• Crises can take many different forms. For instance:
4 www.visionias.in ©Vision IAS
o Balance of payment problems occur when a nation is unable to pay for essential imports or
service its external debt.
o Financial crises stem from illiquid or insolvent financial institutions.
o Fiscal crises are caused by excessive deficits and debt.
• Often, countries that come to the IMF face more than one type of crisis as challenges in one sector spread
throughout the economy. Crises can slow growth, increase unemployment, lower incomes, and create
uncertainty, leading to a deep recession. In an acute crisis, defaults or restructuring of sovereign debt may
be unavoidable.
• IMF lending gives countries breathing room to adjust policies in an orderly manner, paving the way for a
stable economy and sustainable growth. IMF financing facilitates a more gradual adjustment. Because
IMF lending usually is accompanied by a set of corrective policy actions, it signals that appropriate
policies are being put in place, encouraging the return of private investors. IMF lending also aims to
protect the most vulnerable population via policy conditionality. In low-income countries, IMF lending is
also typically meant to catalyze financial support from other donors and development partners.
• Hence option (c) is the correct answer.

Q 8.C
• The Competition Commission of India (CCI) is the chief national competition regulator in India. It is a
statutory body within the Ministry of Corporate Affairs and is responsible for enforcing the Competition
Act, 2002 to promote competition and prevent activities that have an appreciable adverse effect on
competition in India. The CCI looks into cases and investigates them if the same has a negative impact on
competition. CCI also approves combination under the act so that two merging entities do not overtake the
market.
• The central government has given the mandate to check Goods and Services Tax (GST) related
profiteering by businesses to antitrust watchdog Competition Commission of India (CCI) from 1
November 2022, as the term of the National Anti-profiteering Authority (NAA) comes to an end. Hence
option (c) is the correct answer.
• The National Anti-profiteering Authority (NAA) was the statutory mechanism under GST law to check
the unfair profiteering activities by the registered suppliers under GST law. The Authority’s core function
is to ensure that the commensurate benefits of the reduction in GST rates on goods and services done by
the GST Council and of the Input tax credit are passed on to the recipients by way of commensurate
reduction in the prices by the suppliers.
• The Government of India (GoI) introduced the GST Council to modify, regulate and reconcile the goods
and services tax in India. The Council replace all the existing multiple taxation process and introduce new
taxation methods to ease the taxation process for the taxpayers. The Council shall also monitor all the
taxation process to provide support to the respective departments and to avoid the fraudulent process. In
this article, we look into the GST Council and understand one of the key councils that regulate all aspects
of GST in India.
• As per Article 279A of the amended Constitution, the GST Council which will be a joint forum of the
Centre and the States, shall consist of the following members: -
o Union Finance Minister - Chairperson
o The Union Minister of State, in-charge of Revenue of finance - Member
o The Minister In-charge of finance or taxation or any other Minister nominated by each State
Government - Members
• Quality Council of India (QCI) was established as a National body for Accreditation on
recommendations of Expert Mission of EU after consultations in Inter-ministerial Task Force, Committee
of Secretaries and Group of Ministers through a Cabinet decision in 1996. Accordingly, QCI was set up
through a PPP model as an independent autonomous organization with the support of Government of
India and the Indian Industry represented by the three premier industry associations, (i) Associated
Chambers of Commerce and Industry of India (ASSOCHAM), (ii) Confederation of Indian Industry (CII)
and (iii) Federation of Indian Chambers of Commerce and Industry (FICCI).
• QCI is a non-profit organization registered under the Societies Registration Act XXI of 1860. The
Department of Industrial Policy and Promotion, Ministry of Commerce and Industry was designated as
the nodal point for all matters connected with quality and QCI to structure and help implementation of the
Cabinet decision.

5 www.visionias.in ©Vision IAS


Q 9.A
• The Goods Trade Barometer was developed by the World Trade Organization to complement
conventional trade statistics and forecasts. Previously known as the World Trade Outlook Indicator, it
is the world’s leading composite indicator that highlights the turning points in the global merchandise
trade and provides forecasts of its likely trajectory in the near future. Hence option (a) is the correct
answer.
• It is released on a quarterly basis based on the availability of data. It provides real-time data on the
trajectory of merchandise trade relative to the current trends.
• Goods Trade Barometer in its November 2022 release said that the global economy, hit by strong
headwinds and weakening import demand, may see trade growth slow down in the closing months of
2022 and into 2023.

Q 10.C
• The total stock of money in circulation among the public at a particular point of time is called money
supply. RBI publishes figures for four alternative measures of money supply, viz. M1, M2, M3 and M4.
They are defined as follows
o M1 = CU + DD
o M2 = M1 + Savings deposits with Post Office savings banks.
o M3 = M1 + Net time deposits of commercial banks
o M4 = M3 + Total deposits with Post Office savings organisations (excluding National Savings
Certificates)
o Where, CU is currency (notes plus coins) held by the public and DD is net demand deposits held by
commercial banks. The word ‘net’ implies that only deposits of the public held by the banks are to be
included in money supply. The interbank deposits, which a commercial bank holds in other
commercial banks, are not to be regarded as part of money supply.
• M1 and M2 are termed as narrow Money and M3 and M4 are termed as Broad Money.
o These measures are in decreasing order of liquidity. M1 is most liquid and easiest for transactions
whereas M4 is least liquid of all. M3 is the most commonly used measure of money supply. It is also
known as aggregate monetary resources.
o If Rs 50,000 is converted from savings to fixed deposits there will be a decrease in Narrow money.
As Broad Money( M3 and M4) includes M1 plus Time Deposits, the status of Rs 50000 earlier
located in M1 as Demand deposits will now changed to Time deposits i.e reduced from M1 and
added to Time Deposits where both come under M3, which makes no change in total. Just a
difference in the status of money. Hence no change in M3 i.e Broad Money. Hence option (c) is
the correct answer.

Q 11.A
• The Preamble to the Indian Constitution is based on the ‘Objectives Resolution’, drafted and moved by
Pandit Nehru, and adopted by the Constituent Assembly.
• It has been amended by the 42nd Constitutional Amendment Act (1976), which added three new
words–Socialist, Secular, and Integrity.
• The preamble is stated as follows:
o “We, THE PEOPLE OF INDIA, having solemnly resolved to constitute India into a
SOVEREIGN SOCIALIST SECULAR (added through 42nd amendment Act) DEMOCRATIC
REPUBLIC and to secure to all its citizens: Hence option 1 is not correct.
o JUSTICE, Social, Economic, and Political; Hence option 4 is correct.
o LIBERTY of thought, expression, belief, faith, and worship; Hence option 2 is correct.
o EQUALITY of status and of opportunity; and to promote among them all; FRATERNITY assuring
the dignity of the individual and the unity and integrity of the Nation (added through the 42nd
amendment Act); Hence, option 3 is not correct.
o IN OUR CONSTITUENT ASSEMBLY this twenty-sixth day of November 1949, do HEREBY
ADOPT, ENACT AND GIVE TO OURSELVES THIS CONSTITUTION”.

Q 12.D
• Methods of voting in the Lok Sabha: The procedure regarding Voting and Divisions in the House is
governed by Article 100(1) of the Constitution and Rules 367, 367A, 367AA, and 367B of the Rules of
Procedure and Conduct of Business in Lok Sabha.

6 www.visionias.in ©Vision IAS


• The various methods adopted for voting in the Lok Sabha are:
o Voice Vote: It is a simple method for deciding a question put by the Chair on a motion made by a
Member. Under this method, the question before the House is determined by the `Ayes' or the `Noes',
as the case may be. Hence option 1 is correct.
o Division: There are three methods of holding a Division, i.e.
▪ by operating the Automatic Vote Recording Equipment;
▪ by distributing `Ayes' and `Noes' slips in the House; and
▪ by Members going into the Lobbies. However, the method of recording votes in the lobbies
has become obsolete ever since the installation of the Automatic Vote Recording Machine.
o Secret Ballot: During an 'open' voting period, the individual results are shown by the three Colours:
Green for ‘Ayes’, Red for ‘Noes’ and Yellow for ‘Abstain’ on the Individual Result Display Panel.
Secret voting, if any, is held on similar lines except that the Light Emitting Diode (LED) on the
Individual Result Display Panel flashes only white light to show that the vote has been
recorded. Hence option 2 is correct.
o Recording of votes by the distribution of slips: The method of recording of votes by Members on
`Ayes' and `Noes' slips is generally resorted to in the eventuality of
▪ sudden failure of the working of the Automatic Vote Recording Equipment; and
▪ at the commencement of the new Lok Sabha before the seats/division numbers have been allotted
to Members. Hence option 3 is correct.
o A physical count of Members in their places instead of a formal division: If in the opinion of the
Chair, a Division is unnecessarily claimed, he/she may ask the Members who are for `Ayes' and those
for `Noes', respectively, to rise in their places and on a count being taken, he/she may declare the
determination of the House. In such a case, the particulars of the voting of the Members are not
recorded. Hence option 4 is correct.
o Casting Vote: If in a Division the number of `Ayes' and `Noes' is equal, the question is decided by
the casting vote of the Chair. Under the Constitution, the Speaker or the person acting as such cannot
vote in a Division; he/she has only a casting vote which he/she must exercise in the case of equality of
votes.

Q 13.A
• So far, women have held the posts of President of India (Pratibha Devisingh Patil and Draupadi
Murmu) and prime minister (Indira Gandhi) in India, as well as chief ministers of various states
(Sucheta Kriplani, Nandini Satpathy, Anandiben Patel, Sushma Swaraj, etc).
• Since independence, India has had 16 female chief ministers.
o The first woman to become chief minister was Sucheta Kripalani of the Indian National Congress
party, who was sworn in on 2 October 1963 as chief minister of Uttar Pradesh.
• As per the Ministry of Parliamentary Affairs, in the current Lok Sabha (17th), there are 78 female Lok
Sabha members (the number is the highest since Independence). Hence statement 1 is not correct.
• The first female justice in the court was Fathima Bibi appointed on 6 October 1989. There have been
11 female justices in the court since then. Presently there are 3 sitting female judges out of the total
34 judges (including the Chief Justice of India) in the court. Hence statement 2 is correct.
• The Comptroller and Auditor General of India (CAG) is the supreme audit institution of India, established
under Article 148 of the Constitution of India. Since Independence, there have been 14 CAGs in
India, but no woman has ever taken office as a CAG. Hence statement 3 is correct.

Q 14.B
• The states marked in the map given in the question are Arunachal Pradesh, Nagaland, Manipur,
and Mizoram.
• Sixth Schedule: Under Article 244 (2), the provisions of the Sixth Schedule shall apply to the
administration of the tribal areas in the States of Assam, Meghalaya, Tripura, and Mizoram. Hence
option (a) is not correct.
• Inner Line Permit (ILP): Inner Line Permit (ILP) is an official travel document issued by the
Government of India to allow inward travel of an Indian citizen into a protected area for a limited period.
It is obligatory for Indian citizens from outside those states to obtain a permit for entering the protected
state. It is now applicable to the states of Arunachal Pradesh, Nagaland, Manipur, and Mizoram.
Hence option (b) is the correct answer.

7 www.visionias.in ©Vision IAS


• Armed Forces (Assam and Manipur) Special Powers Ordinance was promulgated by the President
on 22nd May 1958. In which some special powers have been given to the members of the armed forces in
disturbed areas in the State of Assam and the Union Territory of Manipur. AFSPA is now applicable fully
only in 31 districts of 4 Northeast states and partially in 12 districts. The four northeastern states
are Nagaland, Assam, Arunachal Pradesh and Manipur. Hence option (c) is not correct.
• At present, India has 28 states and 8 union states. Many Indian states, before attaining statehood, were
union territories. Examples of such states are Himachal Pradesh, Goa, Arunachal Pradesh, Mizoram,
Manipur, and Tripura. Hence option (d) is not correct.

Q 15.B
• The doctrine of sovereign immunity is based on the 'common law' principle borrowed from the
British Jurisprudence that the King commits no wrong and that he cannot be guilty of personal
negligence or misconduct, and as such cannot be responsible for the negligence or misconduct of his
servants. Another aspect of this doctrine was that it was an attribute of sovereignty that a State cannot be
sued in its own courts without its consent. Hence option (b) is the correct answer.
• In the United States, sovereign immunity only applies to the federal and state government and not the
municipalities, as the federal and state government can relinquish their sovereign immunity.
• Meaning of 'common law': The common law is the body of law formed through court decisions, as
opposed to law formed through statutes or written legislation. A common law system is a system of
jurisprudence that is based on the doctrine of judicial precedent, the principle under which the lower
courts must follow the decisions of the higher courts, rather than on statutory laws.

Q 16.A
• In the Golaknath case (1967), the Supreme Court held that Parliament could not amend Fundamental
Rights, and this power would be only with a Constituent Assembly. The Court held that an amendment
under Article 368 is "law" within the meaning of Article 13 of the Constitution and therefore if an
amendment "takes away or abridges" a Fundamental Right conferred by Part III, it is void.
o After the case, the Parliament came up with the 24th Amendment (1971) that explicitly stated
that Article 368 could be used to curtail even fundamental rights. From then on, Article 13 lost
the power to nullify anything written under Article 368. Hence, option (a) is the correct answer.
• In the Kesavananda Bharati case, the Supreme Court overruled its judgment in the Golak Nath
case (1967).
o It laid down a new doctrine of the ‘basic structure’ (or ‘basic features’) of the Constitution.
▪ It ruled that the constituent power of Parliament under Article 368 does not enable it to alter
the ‘basic structure’ of the Constitution.This means that the Parliament cannot abridge or take
away a Fundamental Right that forms a part of the ‘basic structure’ of the Constitution.
• In the Shankari Prasad case (1951), the constitutional validity of the First Amendment Act (1951),
which curtailed the right to property, was challenged.
o The Supreme Court ruled that the power of the Parliament to amend the Constitution under
Article 368 also includes the power to amend Fundamental Rights.
o The word ‘law’ in Article 13 includes only ordinary laws and not the constitutional amendment acts
(constituent laws).
o Therefore, the Parliament can abridge or take away any of the Fundamental Rights by enacting a
constitutional amendment act and such a law will not be void under Article 13.
• In the Minerva Mills case (1980), the Supreme Court provided key clarifications on the interpretation
of the basic structure doctrine.
o The court ruled that the power of the parliament to amend the constitution is limited by the
constitution. Hence the parliament cannot exercise this limited power to grant itself unlimited power.
In addition, a majority of the court also held that the parliament's power to amend is not a power to
destroy.
o The ruling struck down clause 4 and 5 of the Constitution (Forty-second Amendment) Act,
1976.
▪ Clause 4 gave precedence to directive principles of state policy over fundamental rights. Supreme
court struck down this provision and said that harmony and balance between fundamental
rights directive principles is part of basic structure of the constitution.
▪ Clause 5 gave unlimited power to parliament to amend the constitution and this was also
struck down by supreme court deeming it to be destroying the basic structure of the constitution.

8 www.visionias.in ©Vision IAS


Q 17.B
• Scheduled Tribe (ST) status to the Gond community:
o Rajya Sabha passed the Bill to extend ST status to the Gond community in four districts of UP.
o It aims to move the Gond community from the Scheduled Castes list to the Scheduled Tribes list.
• Process of Inclusion
o The process to include tribes in the ST list begins with recommendations from the respective State
governments, which are then sent to the Tribal Affairs Ministry, which reviews and sends them to
the Registrar General of India for approval.
o This is followed by the National Commission for Scheduled Tribes’ approval before the list is
sent to the Cabinet for a final decision.
o The final decision rests with the President (under Article 342). The inclusion of any community
in the STs comes into effect only after the President assents to a Bill amending the Constitution
(Scheduled Tribes) Order, 1950.
o Hence option (b) is the correct answer.
• About Gonds
o The term Gond refers to tribal peoples who live all over India's Deccan Peninsula.
o Most describe themselves as Gonds (hill people) or as Koi or Koitur.
o They live in the states of Madhya Pradesh, Maharashtra, Telangana, Andhra Pradesh, Bihar, and
Odisha.
o The majority speak unintelligible dialects of Gondi which is an unwritten language of the Dravidian
family.
o Persa Pen (divine god or high god) is the most distinctive feature of the Gond religion: Like many
other tribes, Gonds worship a high god known as Baradeo, whose alternate names are Bhagavan, Sri
Shambu Mahadeo, and Persa Pen.

Q 18.B
• A judgment of the Supreme Court becomes the law of the land, according to the Constitution. It is final
because it provides certainty for deciding future cases. However, the Constitution itself gives,
under Article 137, the Supreme Court the power to review any of its judgments or orders. In a 2013
ruling, the Supreme Court itself laid down three grounds for seeking a review of a verdict it has
delivered — the discovery of a new and important matter or evidence which, after the exercise of
due diligence, was not within the knowledge of the petitioner or could not be produced by him;
mistake or error apparent on the face of the record; or any other sufficient reason. In subsequent rulings,
the court specified that “any sufficient reason” means a reason that is analogous to the other two
grounds. Hence statement 2 is correct.
• It is not necessary that only parties to a case can seek a review of the judgment on it. As per the Civil
Procedure Code and the Supreme Court Rules, any person aggrieved by a ruling can seek a review.
However, the court does not entertain every review petition filed. It exercises its discretion to allow a
review petition only when it shows the grounds for seeking the review. A review petition must be filed
within 30 days from the judgment or order of which review is sought and must be placed before the
same Bench which had delivered the decision. Hence, statement 1 is not correct.

Q 19.A
• Recent context: A bill, called the Prohibition of Child Marriage (Amendment) Bill, 2021, was introduced
in the year 2021 and is now under consideration. Also, the Assam government has initiated a crackdown
on all such marriages in which the brides had not attained the age of 18 at the time of marriage. Those
having married girls below 14 years of age will be booked under the Protection of Children from Sexual
Offences (POCSO) Act (2012), and cases under the Prohibition of Child Marriage Act, 2006 will be
registered against those who have married girls in the 14-18 years age group.
• The Prohibition of Child Marriage Act 2006 or PCMA 2006, came into force on 1 November 2007 in
India. To ensure that child marriage is eradicated from within society, the Government of India enacted
the Prevention of Child Marriage Act 2006 by replacing the earlier legislation of the Child Marriage
Restraint Act 1929.
• Important provisions of PCMA:
o An offense punishable under this Act shall be cognizable and non-bailable.
o It also provides for the duties and responsibilities of Child Marriage Prohibition Officers.

9 www.visionias.in ©Vision IAS


o It defines 'male child' and 'female child'; any male person below the age of 21 years and any female
person below the age of 18 years will be considered a child. Hence statement 2 is correct.
o It provides for the maintenance of females after the nullification of marriages.
o This law also prescribes punishment for performing, conducting, and abetting child marriage. Even
the parents are to be punished for promoting or permitting child marriage.
o As per the Act, child marriages are not voidable by default. This confers the status of husband and
wife upon the parties to the marriage and the marriages are not void but merely voidable. Thus such
marriages would continue to be perfectly valid till the time either of the parties chooses to get it
annulled. Hence statement 1 is not correct.
▪ Petition for annulling a child marriage by a decree of nullity may be filed in the district court
only by a contracting party to the marriage who was a child at the time of the marriage.
▪ If at the time of filing a petition, the petitioner is a minor, the petition may be filed through his
or her guardian or next friend along with the Child Marriage Prohibition Officer.
▪ The petition under this section may be filed at any time but before the child filing the petition
completes two years of attaining majority.

o District court shall make an appropriate order for the custody of the children of such child marriage
and while making such order of custody, the court issue order taking in to account of the welfare and
best interest of the child of such marriage. Thus the child's custody will not be transferred to the
mother by default. Hence statement 3 is not correct.

Q 20.C
• The Rajya Sabha has been given four exclusive or special powers that are not enjoyed by the Lok
Sabha:
o It can authorize the Parliament to make a law on a subject enumerated in the State List (Article
249). Hence statement 1 is correct.
o It can authorize the Parliament to create new All-India Services common to both the Centre and states
(Article 312).
o It alone can initiate a move for the removal of the vice president. In other words, a resolution for
the removal of the vice president can be introduced only in the Rajya Sabha and not in the Lok
Sabha (Article 67). Hence statement 2 is correct.
o If a proclamation is issued by the President for imposing a national emergency or president’s rule or
financial emergency at a time when the Lok Sabha has been dissolved or the dissolution of the Lok
Sabha takes place within the period allowed for its approval, then the proclamation can remain
effective even if it is approved by the Rajya Sabha alone (Articles 352, 356 and 360).
• The above points make it clear that the position of the Rajya Sabha in our constitutional system is not as
weak as that of the House of Lords in the British constitutional system nor as strong as that of the Senate
in the American constitutional system. Except in financial matters and control over the council of
ministers, the powers and status of the Rajya Sabha in all other spheres are broadly equal and
coordinate with that of the Lok Sabha.

Q 21.A
• The Batken region, one of Kyrgyzstan's seven regions, is bordered by Tajikistan and Uzbekistan in
the south of the nation. Kyrgyzstan and Tajikistan were engaged in a conflict last year. Kyrgyzstan
said some 137,000 people had been evacuated from conflict areas in Batken, a mountainous region
surrounded to the north, west and south by Tajikistan. Kyrgyzstan reported fighting in its southern Batken
province which borders Tajikistan's northern Sughd region and features a Tajik exclave, Vorukh. The
same area is famous for its jigsaw-puzzle political and ethnic geography and became the site of similar
hostilities the year before, also nearly leading to a war. Hence, pair 1 is correctly matched.

10 www.visionias.in ©Vision IAS


• Ukraine's port city of Odesa was in news recently as it was designated an endangered World
Heritage site by the United Nations cultural agency. UNESCO said its decision recognized "the
outstanding universal value" of the site. But Russia criticized the move, saying it was "politically
motivated". Known as the pearl of the Black Sea, Odesa has been bombed several times since Russia
invaded Ukraine. Hence, pair 2 is not correctly matched.
o It stands on a shallow indentation of the seacoast about thirty kilometres north of the Dniester River
estuary.
o The city was founded in 1794 by a strategic decision of the Empress Catherine II to build a warm-
water port following the conclusion of the Russo-Turkish war of 1787-1792.
o The new city, built on the site of a Turkish fortress, was initially planned by a military engineer and
then expanded further during the 19th century.

• Kamchatka is home to 29 active volcanoes, part of a vast belt of Earth known as the “Ring of Fire”
which circles the Pacific Ocean and is prone to eruptions and frequent earthquakes. According to
scientists, The Shiveluch volcano in Russia’s far eastern Kamchatka peninsula may be gearing up for its
first powerful eruption in 15 years. Shiveluch is one of the largest and most active volcanoes in
Kamchatka, having erupted at least 60 times in the past 10,000 years. The volcanoes of Kamchatka are
also designated as World Heritage sites. It is located in Russian Far East. Hence, pair 3 is not
correctly matched.

11 www.visionias.in ©Vision IAS


Q 22.C
• The United Arab Emirates (UAE) became a constitutional federation in 1971. Since then it has grown into
one of the world’s richest federations off the back of lucrative oil exports.
• The UAE consists of seven emirates; each has its own leader (Emir) and possesses the powers to
manage its own resources autonomously. The seven emirates are as follows: Abu Dhabi, Dubai, Sharjah,
Ras Al Khaimah, Ajman, Umm Al Quwain, and Fujairah. Hence statement 1 is correct.
• The seven Emirs make up the Federal Supreme Council (FSC), which selects the UAE’s President and
cabinet. The FSC is the UAE’s highest authority, holding legislative and executive powers. Hence
statement 2 is correct.
• Below the FSC is the Federal National Council (FNC), which consists of 40 members with four year
terms. Half of the members of the FNC are appointed by the President and the other half are elected by the
people; each emirate has a specific number of representatives depending on its size. The council has the
ability to pass or reject federal bills, examine the Annual General Budget.
• There are no political parties in the UAE, however, the political involvement and engagement of
citizens is developing. The election of the President and Vice President occurs every five years; however
only the FSC takes part in the poll, so the public doesn’t get a say.
• India and UAE have shared trade links through the centuries. The trade, which was dominated by
traditional items such as dates, pearl and fishes, underwent a sharp change after the discovery of oil in
UAE (oil exports begun from Abu Dhabi in 1962). With the emergence of UAE as a unified entity in
1971, exports from India started growing gradually over the years.
o India-UAE trade, valued at US $ 180 million per annum in the 1970s, is today US $ 59 billion making
UAE, India's third largest trading partner for the year 2019-20 after China and US.
o UAE is the third largest export destination of India (after US and China) with an amount of nearly US
$ 16 billion for the year 2020-21.
o For UAE, India is the third largest trading partner for the year 2020 with an amount of around US $
27.93 billion (non-oil trade).
o The UAE’s investment in India is estimated to be around US $ 17-18 billion of which US 11.67
billion is in the form of FDI while the remaining is portfolio investment.
• India-UAE Agreement for Cultural Cooperation, signed on 3rd January 1975, is the basis for bilateral
cultural cooperation.
o Indian expatriate community of approximately 3.4 million is the largest ethnic community in UAE
constituting roughly about 35 per cent of the country’s population. 15% of the diaspora is in the
Emirate of Abu Dhabi and the rest in 6 Northern Emirates including Dubai. Hence statement 4 is not
correct.

12 www.visionias.in ©Vision IAS


• In 2021, the US was the world's largest producer of crude oil. Russia became the world’s second-
largest crude oil producer, surpassing Saudi Arabia’s output by around 1.5%. Thus, it was Saudi
Arabia which was the third largest producer of crude oil in the world in the year 2021 and not the
UAE. Hence statement 3 is not correct.
• Top crude oil producing countries in the world:

All above data in Unit: Mt

Q 23.D
• Recent Context: Water-sharing principles were in the news in the context of water-sharing between India
and Pakistan in the Indus Water Treaty, 1960.
• Also, with regard to the water sharing principles, the February 16, 2018, judgment of the Supreme
Court relating to Cauvery water dispute is a landmark one in the context of water resource
management, especially in resolving any water-sharing dispute.
o The judgment is the culmination of a long-drawn litigation against the Cauvery Tribunal award 2007
by states/UT such as Tamil Nadu, Karnataka, Kerala and Puducherry.
o For the first time, the apex court has set the principles of water sharing among States for guidance to
policy-makers and water managers. First, the apex court invoked certain settled international
principles for adoption in any water-sharing discourse. It cited four principles in this regard —
Harmon doctrine, Helsinki Rules, Campione Rules and Berlin Rules.
• International water law (also known as international watercourse law, or international law of water
resources) is a term used to identify those legal rules that regulate the use of water resources shared
by two or more countries. The primary role of international water law is to determine a state’s
entitlement to the benefits of the watercourse (substantive rules) and to establish certain requirements for
states’ behavior while developing the resource (procedural rules).
o International water law:
▪ The Harmon doctrine: It postulates that every state is sovereign in its water resources and has
the right to do whatever it likes within its territorial jurisdiction.
▪ Campione Rules: They outline the need to include the water of an aquifer (that is, underground
water or fossil water) while determining a reasonable equitable share.
▪ Helsinki rules, 1996: It established the principle of “reasonable and equitable utilization” of the
waters of an international drainage basin among the riparian states as the basic principle of
international water law.
▪ Berlin Rules 2004: It provides that basin states should manage the water of an international
drainage basin having due regard for the obligation not to cause significant harm to other basin
states.
• Hence, option (d) is the correct answer.

13 www.visionias.in ©Vision IAS


Q 24.D
• The solar wind is a gas of charged particles known as plasma, a state of matter governed by its own set of
physical laws just as the more common solids, liquids, and gases are. As the solar wind sweeps out into
space, it creates a space environment filled with radiation as well as magnetic fields that trail all the way
back to the sun.
• The solar wind is different from winds in the lower atmosphere because it is always blowing. All of the
planets immersed in the solar coronal atmosphere are interacting with it all the time. The electromagnetic
(EM) interaction induces large-scale currents and forms magnetic cavities around magnetized planets.
These cavities are called magnetospheres. Except for Mars and Venus, which do not have intrinsic
magnetic fields, the planets in our solar system all have magnetospheres.
o The magnetosphere is the region above the ionosphere that is defined by the extent of the Earth’s
magnetic field in space.
o The outer boundary of the magnetosphere is called the magnetopause and it separates the
domains of the planetary magnetic field and the solar wind that blows outside it. Its location is
determined by the pressure balance between the solar wind and the planetary magnetic field.
o On the Sun-facing side of Earth – where the magnetic field is compressed by the constant
bombardment of the solar wind – the magnetosphere extends some 6 to 10 times the radius of
Earth. The side of the magnetosphere facing away from the Sun – the nightside – stretches out
into an immense magnetotail, which fluctuates in length and can measure hundreds of Earth
radii, extending far past the Moon's orbit at 60 Earth radii. Hence option (d) is the correct
answer.
• Periods of intense activity, called geomagnetic storms occur when a coronal mass ejection
erupts from the Sun and sends a shock wave through the Solar System. It takes just two days to reach the
Earth.
o At the Earth’s surface, a magnetic storm is seen as a rapid drop in the Earth’s magnetic field
strength.
o Ring Current: Ring current is the name given to the large electric current that circles the Earth above
its equator during magnetic storms.

• The Van Allen radiation belt is a zone of energetic charged particles, most of which originate from the
solar wind. The particles are captured by and held around a planet by that planet’s magnetic field. It
surrounds Earth, containing a nearly impenetrable barrier that prevents the fastest, most energetic
electrons from reaching Earth. The outer belt is made up of billions of high-energy particles that originate
from the Sun and become trapped in Earth’s magnetic field, an area known as the magnetosphere. The
inner belt results from interactions of cosmic rays with Earth’s atmosphere.
• The Van Allen belts are most intense over the Equator and are effectively absent above the poles.
No real gap exists between the two zones; they actually merge gradually, with the flux of charged
particles showing two regions of maximum density.

14 www.visionias.in ©Vision IAS


Q 25.D
• The International Date Line (IDL), established in 1884, passes through the mid-Pacific Ocean and
roughly follows a 180 degrees longitude north-south line on the Earth. It is located halfway around the
world from the prime meridian — the 0 degrees longitude line in Greenwich, England. Hence statement
1 is not correct.
• The IDL functions as a “line of demarcation” separating two consecutive calendar dates.
• When one crosses the date line, one becomes a time traveler of sorts. For instance, cross to the west and
it’s one day later; cross back and you’ve “gone back in time."
• The line of longitude 180° is one and the same for East or West of the Prime Meridian. Since it is
diametrically opposite to the Greenwich Meridian it causes a time difference of a full day on crossing the
line.
o A traveler going eastwards gains time from Greenwich until he reaches the meridian 180°E, when he
will be 12 hours ahead of G.M.T. Similarly in going westwards, he loses 12 hours when he reaches
180°W. There is thus a total difference of 24 hours or a whole day between the two sides of the 180°
meridian. This is the IDL where the date changes by exactly one day when it is crossed.
o When crossing this date line from the west to east, a day is deducted or the time is fixed backward
by one day i.e a day will be gained.
o When crossing the IDL from east to west, a day is added, or the calendar is fixed one day ahead i.e. a
day will be lost. Hence statement 3 is not correct.
• Despite its name, IDL has no legal international status and countries can choose the dates they observe.
While the date line generally runs north to south from pole to pole, it zigzags around political borders
such as eastern Russia and Alaska’s Aleutian Islands. Hence statement 2 is correct.

Q 26.A
• Indian mangoes come in various shapes, sizes and colors with a wide variety of flavors, aromas and tastes.
The Indian mango is a special product that substantiates the high standards of quality and bountiful
nutrients packed in it. A single mango can provide up to 40 percent of the daily dietary fiber needs - a
potent protector against heart disease, cancer and cholesterol build-up. In addition, this luscious fruit is a
warehouse of potassium, beta- carotene and antioxidants. In India, mangoes are mainly grown in tropical
and subtropical regions from sea level to an altitude of 1,500m. Mangoes grow best in temperatures
around 27℃.
• India is the home of about 1,000 varieties. However, only a few varieties are commercially cultivated
throughout India. Most Indian mango varieties have specific eco-geographical requirements for optimum
growth and yield. The Northern/Eastern Indian varieties are usually late bearing compared to
Southern and Western Indian varieties. Some of the local varieties of mango bear fruits throughout
the year in extreme southern parts of India. Hence statement 1 is correct.
• The major mango-growing states are Andhra Pradesh, Uttar Pradesh, Karnataka, Bihar, Gujarat and
Telangana. Uttar Pradesh ranks first in mango production with a share of 23.58 % and the highest
productivity in 2021-22. Hence statement 2 is correct.
15 www.visionias.in ©Vision IAS
• The major varieties of mango are given in the table along with the region of occurrence:

• The Suvarnarekha variety is found in Andhra Pradesh. Hence statement 3 is not correct.

Q 27.D
• Kakrapar nuclear power plant: It is a nuclear power plant in India, which lies in the proximity of the
Mandvi, Surat and Tapi rivers in the state of Gujarat.
• Tarapur nuclear power plant: It is located near Boisar, Maharashtra and is the oldest nuclear power
plant in India. It completed 50 years of operation in October 2019.
• Kaiga nuclear power plant: Kaiga Generating Station is a nuclear power generating plant situated in
Karnataka. The plant has been in operation since March 2000 and is operated by the Nuclear Power
Corporation of India.
• Kalpakkam nuclear power plant: It is located at Kalpakkam near Chennai in the state of Tamil Nadu.
It is also called Madras Atomic Power Station. It is also India's first fully indigenously constructed
nuclear power station.
• Kudankulam nuclear power plant: It is located south of Chennai. Two 1,000-megawatt (MW)
pressurized water reactor (PWR) units based on Russian technology were constructed in phase one of the
project. An additional four units are scheduled to be added according to the agreement signed between
India and Russia in December 2008.
• Above all nuclear power plants were developed and Operated by the Nuclear Power Corporation of India
(NPCIL).
• The correct sequence from north to south is 1-5-4-3-2. Hence, option (d) is the correct answer.

16 www.visionias.in ©Vision IAS


Q 28.C
• Recently, a Spicejet aircraft was caught in Kalbaishakhi while landing, and 10 were injured as bags
rained on flyers. The problem occurred due to air turbulence by norwesters locally called
Kalbaishakhi.
• Violent thunderstorms in the Gangetic plains of India are locally known as Kal Baisakhi or Nor'westers.
These localized events are generally associated with thunderstorms accompanied by strong squally winds
and torrential rainfall. Kal Baisakhi definitely brings destruction in terms of lightning, thunderstorm,
hailstorm, and rainfall. Kalbaisakhi is a common occurrence during April and May. Due to this,
planes face heavy turbulence and uneven landing. Hence, option (c) is the correct answer.
• All of Western and Central Asia remains under the influence of westerly jet streams along an
altitude of 9-13 km from west to east in the winter season. These winds blow across the Asian
continent at latitudes north of the Himalayas roughly parallel to the Tibetan highland. Hence, option (b)
is not correct.
• An easterly jet stream flows over the southern part of the Peninsula in June and has a maximum
speed of 90 km per hour. In August, it is confined to 15 degrees North latitudes, and in September up to
22 degrees N latitudes. The easterlies normally do not extend to the north of 30 degrees N latitude in the
upper atmosphere. Hence, option (a) is not correct.
• The Bay of Bengal branch of the Monsoon reaches Assam in the first week of June and gets deflected
towards the West by the mountain ranges, thus giving rainfall to the Ganga Plains. The Northeast
monsoon also called as 'retreating monsoon' completes the monsoon season starting in the month of
November and ending in the month of December. Hence, option (d) is not correct.

Q 29.B
• The Nile River flows from south to north through eastern Africa. It begins in the rivers that flow into
Lake Victoria (located in modern-day Uganda, Tanzania, and Kenya), and empties into the Mediterranean
Sea more than 6,600 kilometers (4,100 miles) to the north, making it one of the longest river in the
world. The Nile River flows over 6,600 kilometers (4,100 miles) until emptying into the
Mediterranean Sea. Hence, pair 1 is not correctly matched.

17 www.visionias.in ©Vision IAS


• The Amazon River is a massive, intricate water system weaving through one of the most vital and
complex ecosystems in the world- the Amazon rainforest in South America. It is by far the mightiest
river on Earth in terms of volume and width. Its westernmost source is high in the Andes Mountains,
within 160 km of the Pacific Ocean, and its mouth is in the Atlantic Ocean, on the northeastern coast
of Brazil. Hence, pair 2 is correctly matched.

• Yangtze River is the longest river in both China and Asia and the third longest river in the world,
with a length of 6,300 km. From its source on the Plateau of Tibet to its mouth on the East China
Sea, the river traverses or serves as the border between 10 provinces or regions. More than three-fourths
of the river’s course runs through mountains. Hence, pair 3 is not correctly matched.

18 www.visionias.in ©Vision IAS


• Danube River is the second-longest river in Europe. It rises in the Black Forest mountains of
western Germany and flows for some 2,850 km to its mouth on the Black Sea. Along its course, it
passes through 10 countries: Germany, Austria, Slovakia, Hungary, Croatia, Serbia, Bulgaria, Romania,
Moldova, and Ukraine. It is recently in news along with many other important rivers of Europe because of
extensive drying up. After years of dry weather, scientists are warning that low-water conditions could
become the norm in Europe as the climate changes. Hence, pair 4 is correctly matched.

Q 30.A
• Tropical cyclones are rapidly rotating violent storms that originate over tropical oceans in late summers.
• They are characterized by:
o a closed low-pressure centre with steep pressure gradients, which is responsible for the wind speeds,
o a closed low-level atmospheric circulation (cyclonic circulation) - a result of rapid upward movement
of hot moist air which is subjected to Coriolis force,
o strong winds (squalls - a sudden violent gust of wind), and
o a spiral arrangement of thunderstorms that produce very heavy rain (torrential rainfall)
• Conditions necessary for the formation of a tropical cyclone:
o Large sea surface with a temperature higher than 27°C.
▪ Ocean waters having temperatures of 27° C and depth of warm water extending for 60-70 m deep
supply enough moisture, and hence latent heat of condensation, to generate and drive a tropical
storm. The thick layer of warm water ensures that the deep convection currents within the water
do not churn and mix the cooler water below with the warmer water near the surface.
19 www.visionias.in ©Vision IAS
▪ The tropical cyclone derives its energy from the latent heat of condensation, and the
difference in densities of the air masses does not contribute to the energy of the
cyclone. Whereas, the energy of a temperate cyclone depends on the temperature, humidity
and density differences of air masses. Hence statement 1 is not correct.
▪ During strong El Nino years, strong hurricanes occur in the eastern Pacific. This is due to the
accumulation of warm waters in the eastern Pacific due to the weak Walker Cell. Hence
statement 2 is correct.
o Presence of the Coriolis force enough to create a cyclonic vortex.
o A pre-existing weak low-pressure area or low-level cyclonic circulation
o Low wind shear:
▪ Vertical Wind Shear is the difference between wind speeds at different altitudes. The larger the
change in wind speed and direction with the altitude, the stronger the vertical wind
shear. Tropical cyclones develop when the vertical wind shear is low. High wind shear
inhibits the development of towering cumulonimbus clouds and the cyclonic vortex. Hence
statement 3 is not correct.
▪ If a tropical cyclone is being affected by rather strong vertical wind shear for a sustain
period of time, the storm structure will be damaged and its low-level circulation could be
separated from the main convective cluster, further weakening the tropical cyclone and even
possibly causing it to dissipate.

Q 31.B
• Lord Stanley the President of the Board of Control introduced a bill for the 'Better Government' of
India which became an Act of Parliament in August 1858. Hence, the act was also called the Act for
Better Government of India.
o It transferred the power to govern from the East India Company to the British Crown.
o It replaced the President of the Board of Control with a Secretary of State for India, who became
"in subordination to the cabinet, the fountain of authority as well as the director of policy in India".
He was to be advised by a Council of India, consisting of fifteen members, seven of whom were to
be selected from the now superseded Court of Directors.
o As per the act of 1858, the Governor General of India was given the additional title of the
Viceroy. He retained all his powers but instead of earlier dual control, he was now made answerable
only to the secretary of state. Hence statement 2 is correct.
• It was the Government of India Act of 1919, that provided for a High Commissioner for India, who
was to hold his office in London for six years. Hence statement 1 is not correct.

Q 32.C
• The ‘Case of Nand Kumar’ (1775) during the reign of Warren Hastings (1772-1785) raised concerns
about the applicability of English law in India. Hence option 3 is correct.
o Nand Kumar was an influential official in Bengal. He was hanged to death by the verdict of the
Supreme Court at Calcutta for a petty offense of forgery. English law was applied in this judgment. It
was contended that Warren Hastings and Sir Elija Impey, the judge of the Supreme Court conspired
against Nanda Kumar.
o Raja Nand Kumar brought several charges against then Governor-General Warren
Hastings. The charges were related to the offenses of bribery and corruption, after which he
himself was accused and convicted of forgery and became the first person in India to be
executed by hanging.
▪ In 1787, Warren Hastings was impeached in the Parliament by Edmund Burke and the
Whigs for his administrative excess. The most important of charges against him were related to
the Rohilla War, the Case of Nanda Kumar, the treatment of Raja Chait Singh of Benares and the
pressures on the Begums of Oudh.
• The case of Nanda Kumar thus, was not linked to the issue of Sati or Religious
conversion. Hence options 1 and 2 are not correct.

Q 33.C
• Farrukh Siyar's Farman (1717) was a grant giving concessions to the English east india company
and allowing them tax-free monopoly trade specially in the Mughal subah of Bengal, Bihar and
Orissa. It was the outcome of prolonged efforts by the company with the sole objective of avoiding the
payment of usual taxes and tolls. Hence statement 1 is correct.
20 www.visionias.in ©Vision IAS
• Farrukh Siyar's Farman of 1717 relating to the Subah of Bengal included the privileges listed
below:
o That all the goods and necessaries carried by the agents of the English company either by land or
water would be free of custom-duties on payment of the yearly peshkash of Rs.3000.
o If the goods of the company be stolen, every measure should be taken to recover the lost goods and
punish the thief.
o In their attempt to establish factories at any place, they should be provided with every assistance.
o In the event of any native merchant or weaver becomes indebted to the company's factors (agents), the
amount should be paid back.
o Measures should be taken so that the boats owned or hired by them are not molested by anyone.
o That the villages bought by the company remain in their possession and the Diwan of the subah
shall accord permission for renting of some adjoining villages. Hence statement 2 is correct.
o If the company's goods are lost in shipwrecks, special care should be taken of the belongings of the
company.
• In addition to the above important privileges granted by the farman of the emperor, the hasb al-hukum
issued by the prime minister, contained some additional privileges such as
o Issuing of dastak (pass) by the chief of the factory, which would ensure the free passage of goods
under the name of the English company without being checked at the customhouses
o The coining of the company's gold and silver in the Murshidabad mint may be allowed for three days
a week if it is not against the interest of the nawab
o The possession of the villages with zamindari rights in and around Calcutta along with the permission
of farming some other villages petitioned for should be allowed with the permission of the diwan of
the Subah.

Q 34.B
• Mahatma Gandhi announced his plan to begin Non-Cooperation with the government as a sequel to the
Rowlatt Act, the Jallianwala Bagh massacre, and the Khilafat Movement. On August 31, 1920, the
Khilafat Committee started a campaign of non-cooperation and the movement was formally
launched. It was approved by the Indian National Congress at the Nagpur session in December 1920.
o The movement began with Mahatma Gandhi renouncing the titles, which were given by the
British.
o Other leaders and influential persons also followed him by surrendering their honorary posts
and titles.
o Students came out of the government educational institutions.
o National schools such as the Kashi Vidyapeeth, the Bihar Vidyapeeth, and the Jamia Millia
Islamia were set up. Hence option (b) is the correct answer.
• Babu Shiv Prasad Gupt and Bhagwan Das established the university in Varanasi, on 10 February
1921, during the non-cooperation movement of the freedom struggle. Originally named Kashi
Vidyapeeth, the university was renamed Mahatma Gandhi Kashi Vidyapeeth in 1995. It
was inaugurated by Mahatma Gandhi.
• Mahatma Gandhi laid the foundation of Bihar Vidyapeeth on February 6, 1921, on a sprawling
campus in western Patna. The institute came up on land donated by eminent educationist, freedom fighter
and lawyer Maulana Mazaharul Haq.
• Jamia Millia Islamia was established at Aligarh in United Provinces in 1920. On 22 November 1920,
Hakim Ajmal Khan was elected the first chancellor of Jamia.

Q 35.C
• Shyamji Krishna Varma (4 October 1857 – 30 March 1930) was an Indian revolutionary fighter, an Indian
patriot, lawyer and journalist.
• He founded the Indian Home Rule Society, India House and The Indian Sociologist in London.
• In 1905, he founded the India House and The Indian Sociologist, which rapidly developed as an
organised meeting point for radical nationalists among Indian students in Britain at the time.
• It was one of the most prominent centres for revolutionary Indian nationalism outside India.
• Krishna Varma later moved to Paris in 1907, avoiding prosecution.
• India House:
o As many Indian students faced racist attitudes when seeking accommodations, he founded India
House as a hostel for Indian students, based at 65, Cromwell Avenue, Highgate.

21 www.visionias.in ©Vision IAS


o This living accommodation for 25 students was formally inaugurated on 1 July by Henry Hyndman,
of the Social Democratic Federation, in the presence of Dadabhai Naoroji, Lala Lajpat Rai, Madam
Cama etc.
o Shyamji hoped India House would incubate Indian revolutionaries and Bhikaiji Cama, S. R. Rana,
Vinayak Damodar Savarkar, Virendranath Chattopadhyaya, and Lala Hardayal were all associated
with it.
o It was opened to promote nationalist views among Indian students in Britain. Hence, option (c)
is the correct answer.
o This institute used to grant scholarships to Indian youths for higher studies in England. The
building rapidly became a hub for political activism, one of the most prominent for overseas
revolutionary Indian nationalism.
o "India House" came to informally refer to the nationalist organisations that used the building at
various times.
o Shyamji's activities in England aroused the concern of the British government:
o He was severely criticised for writing anti-British articles in The Indian Sociologist.
o Most of the British press were anti–Shyamji and printed several allegations against him and his
newspaper.

Q 36.D
• Gautama Buddha was the noble prince of a Sakya clan, whose father ruled in Kapilavastu.
• He was born in Lumbini, lived in a palace, married, had a young son, and then, gave it all up, to seek the
real meaning of Life.
• Bodhgaya: For six years, he wandered as a mendicant and finally came to the village of Uruvela (now,
Bodhgaya), on the banks of the river Nairanjana (now, the Phalgu river). Here, he devoted himself to
severe penance and meditation and withstood many trials and tribulations. After 49 days, Siddhartha
gained the knowledge he was seeking and became the Buddha. He became the Enlightened One.
o The Bodhi Tree is the most important attraction in the temple complex. It has been transplanted at
least five times, at Bodhgaya. The present one is supposed to have come from a sapling brought by
Emperor Ashoka.
▪ Under the Bodhi tree, is the Vajrasana, a rich, red slab of stone which had been placed by
Emperor Ashoka, to denote the exact spot where the Buddha had gained
enlightenment. Made of sandstone, the Vajrasana features animal and bird motifs at the side
and carved geometrical patterns on the surface.
o Other famous places at Bodhgaya: There’s the Animesh Lochan Chaitya, where the Buddha gazed at
the Bodhi tree without blinking, the Ratnachakrama or Jewelled path, where lotus flowers bloomed
under the Buddha’s feet;
▪ Ratnaghar Chaitya, where the rays of five colours emanated from the Buddha’s body, and later
became the colours connected with the religion (blue, red, yellow, white, orange).
▪ There are also the Rajayatna Tree and the Ajapala Nigrodlha Tree, where the Buddha
meditated.
▪ Muchchalinda pond, where he meditated during the rains, arid was protected by the hood of the
Nagraj serpent .especially when Mara, the demon, created thunder, lightning, storms, to distract
him, and also sent his three daughters to tempt him.
▪ The Mahabodhi Temple was first built by Emperor Ashoka in the 3rd century BC, and the
present temple dates from the 5th century or 6th century AD. It is one of the earliest Buddhist
temples built entirely in brick, still standing in India. It is also UNESCO's world heritage site.
• Sarnath is a very important Buddhist spot, because this is where the Buddha gave his first sermon (at
Deer Park), called the Dharmachakraparavartana or the Wheel of Law, five weeks after his
enlightenment.
o It was here that he propagated the important tenets of his Dharma - the Four Noble Truths, the
Eight-Fold path, and the Middle Way. It was here also that he founded his Sangha (Council),
with five of his former disciples.
o The Triple Gem concept of Buddhism originated here (Buddha, Dharma, Sangha), as also the
celebrated mantra ‘Buddham Sharanam, Gachami.’
o The Chaukhandi Stupa, on a high mound, is one of the first sights in Sarnath. It has the ruins of a
Buddhist monastery of the 5th century AD, supposedly to mark the spot where Buddha first met
his five disciples, when he reached Sarnath.

22 www.visionias.in ©Vision IAS


o It was Emperor Ashoka, who resurrected this sacred Buddhist town, when he visited it and erected
famous Ashoka Pillar. The four lion-heads on top of the pillar, facing four different directions and
called the Lion Capital, is on display in the Sarnath Museum. He also constructed the biggest and
most striking stupa, the Dhamek Stupa.
o There is also the Dharmarajika Stupa, which is one of the few pre-Ashokan stupas in
existence; Mulgandhakuti Vihara, where the Buddha meditated, whenever he visited Sarnath,
during the monsoons.
• Rajgir, formerly known as Rajagaha, was the first capital of Magadha, ruled by King Bimbisara who
was one of the most prominent supporters of the Buddha. Buddha used this city as his monsoon retreat,
and preached his Dharma.
o Buddha preached his sermons from the striking - looking Gridhakuta Hill or Vulture Peak,
which had a unique rock formation, like a vulture’s beak.
o The “Bimbisrara Jail” has been excavated in Rajgir, as also the Ajatashatru Fort, where his son
Ajatashatru repented, after killing his father, converted to a Buddhist, and placed a statue of Buddha
in the ‘Parinirvana’ pose.
o The Venuvana Vihara or the Monastery of the Bamboo Grove, was Bimbisara’s first offering to
the Buddha.
o Raigir also boasts of hot sulphur springs, Satadhara, which supposedly have healing powers and
which are now part of the Hindu Lakshminarayana Temple complex.
o Other interesting sites in Rajgir, include an aerial ropeway which takes visitors up the hill to the
gorgeous Vishwa Shanti Stupa built by the Japanese.
o There are also the grand Pippala Caves and the seven Sataparni Caves, where the first Buddhist
Council was said to have been held, after the Buddha’s Mahaparinirvana.
• Sravasti was the capital of the ancient kingdom of Kosala, and the biggest town in the Gangetic plains
during the Buddha’s time.
o Here, Buddha stayed in the Jetvana Vihara, Anandakuti and Gandhakuti, whose ruins still remain,
and are sacred to the Buddhist pilgrims.
o Also found here is Anand Bodhi tree, planted by Buddha’s favourite disciple, Anand.
o There is also the Angulimala Stupa, built in memory of the killer- dacoit Angulimala, notorious
for wearing a necklace of human fingers around his neck. When he met the Buddha, the latter’s
serene words changed him, and he became an ardent disciple.
• Hence option (d) is the correct answer.

Q 37.A
• Akbar was born on 25 October 1542 in Amarkot. He was the third Mughal emperor, who reigned from
1556 to 1605.
• Goswami Tulsidas (1532–1623) was a contemporary of Akbar (1542-1605) and wrote ‘Ramcharit
Manas’ in the Avadi language. According to accounts of Priyadas, Tulsidas met the emperor Akbar and
later became friends with him. (NOTE: Different historical pieces of evidence suggest a different birth
year of Tulsidas, but certainly, he was a contemporary of Akbar)
• Kabir Das (1398/1440-1512) and Vallabhacharya (1479-1531) were 15th-century Indian saints and pre-
date Akbar.
• Other contemporaries of Akbar were Tansen and Abul fazl.
• Hence, option (a) is the correct answer.

Q 38.B
• The Arthashastra is an Indian treatise on politics, economics, military strategy, the function of the
state, and social organization attributed to the philosopher and Prime Minister Kautilya.
• The Arthashastra advocated strict control of gambling, which it would confine entirely to officially-
managed gambling houses, financed by a tax of five percent of the stakes and a charge for the hire of dice
to gamblers, who were to be forbidden to use their own. Stringent fines were laid down for
cheating. Hence statement 1 is correct.
• The Arthashastra advises the manufacture of liquor in government breweries and distilleries and
gives several briefs and cryptic recipes, which shows that there were many alcoholic drinks, among these
were rice beer (medaka), a sort of spiced beer made of flour (Prasanna), wood-apple wine (asava),
maireya, a liquor made of raw sugar. Hence statement 2 is correct.

23 www.visionias.in ©Vision IAS


o Wine from grapes was made in the North-West and exported in small quantities to the rest of India.
o In the South, toddy, the fermented sap of the palmyra or coconut, was the staple alcoholic liquor, and
is frequently mentioned in early Tamil literature.
o The Arthashastra, perhaps reflecting Mauryan conditions, suggests the appointment of a
"superintendent of liquor", not only to control the sale and consumption of alcoholic drinks but to
organize their manufacture. The text advises the stringent control of private brewing, and of taverns.
Tavern-keepers were to be instructed to make their establishments well-furnished and comfortable and
to prevent their patrons from over-indulgence.
• Arthashastra regulated prostitution. The state also enabled the setting up of establishments with lump-
sum grants to the head courtesan and presumably to enable them to buy jewelry, furnishings, musical
instruments, and other tools of their trade. Hence statement 3 is not correct.
o Independent prostitutes, who were neither given a grant nor required to produce detailed accounts,
had to pay a tax of one-sixth of their income.

Q 39.B
• The taxes during the Mauryan period were directly collected by the king’s officials from the individual
cultivators without bringing in intermediaries. In addition, the Arthasastra states that the amount of tax
would also depend on the nature of irrigation facilities and would range from one-fifth to one-third.
The Rummindei inscription mentions that the king deals directly with the question of exemption
from land tribute.
o The village that was exempted from taxation was called parikrama. Hence statement 3 is not
correct;
o those that supplied soldiers, ayudhiya;
o those that paid their taxes in the form of grain, cattle, gold, or raw material was called kupya.
o There were also the villages that supplied free services and dairy produce in lieu of taxes.
• The Mauryan period witnessed a rapid development of material culture in the Gangetic
plains. Given the access to the rich iron ores of south Bihar, people used more and more of this metal.
This period evidences socketed axes, hoes, spades, sickles, and plowshares. Besides these iron
implements, the spoked wheel also began to be used. Although arms and weapons were the monopoly
of the Maurya state, the use of the other iron tools was not restricted to any class. Hence statement 1
is not correct.
• In the end of the Maurya period burnt bricks were used in north-eastern India. Maurya
constructions made of burnt bricks have been found in Bihar and UP. Houses were made of bricks, and
also timber which was available in abundance because of the dense vegetation in ancient times. Hence
statement 2 is correct.

Q 40.D
• Jainism: Jainism is an ancient religion that is rooted in the philosophy that teaches the way to liberation
and a path to spiritual purity and enlightenment through disciplined nonviolence to all living creatures.
• Jainism came to prominence in the 6th century B.C., when Lord Mahavira propagated the religion.
o There were 24 great teachers, the last of whom was Lord Mahavira.
o These twenty-four teachers were called Tirthankaras-people who had attained all knowledge
(Moksha) while living and preached it to the people.
o The first Tirthankara was Rishabnatha.
o The word ‘Jain’ is derived from Jina or Jaina which means the ‘Conqueror’.
• Jainism was patronised in North India by:
o Nandas, Bimbisara, Ajatashatru and Udayin (Haryanka dynasty), Chandragupta Maurya,
Bindusara and Samprati (Magadhan dynasty)
o Pradyota (Avanti)
o Kharavela (Kalinga)
• Jainism was patronised in South India by:
o Ganga dynasty
o Kadamba dynasty
o Amoghavarsha (Rashtrakuta dynasty)
o Siddharaj Jai Singh and Kumarpala (Chalukya/Solanki)- were the last great patrons of Jainism.
• Hence, option (d) is the correct answer.

24 www.visionias.in ©Vision IAS


Q 41.A
• Edge species" refers to species that thrive in habitats that are located on the edges of two different
ecosystems, such as forest edges, meadow edges, or water edges
o Edge species are adapted to thrive in the transition zones between different ecosystems.
o Edge habitats can provide more resources and opportunities for edge species than interior habitats.
o Some edge species have unique adaptations that allow them to thrive in both ecosystems, such as the
ability to tolerate different soil types or weather conditions.
• The EDGE of Existence programme is a conservation programme of the Zoological Society of London
(launched in January 2007), and is the only global conservation initiative to focus specifically on threat-
ened species that represent a significant amount of unique evolutionary history.
o Using a scientific framework to identify the world’s most Evolutionarily Distinct and Globally Endan-
gered (EDGE) species, the EDGE of Existence programme highlights and protects some of the weird-
est and most wonderful species on the planet.
• Edge species are often more susceptible to invasive species and other threats than species that are
adapted to interior habitats. This is because edge species have evolved to thrive in the unique
conditions of transitional zones, which can be disrupted by invasive species, habitat fragmentation, or
other disturbances.
o Invasive species, for example, can easily invade edge habitats and outcompete native edge species for
resources such as light, water, and nutrients. This can lead to declines in the abundance and diversity
of native edge species, which can have cascading effects on ecosystem functioning.
• Hence option (a) is the correct answer.

Q 42.C
• India is one of the few countries to have a scientific system of periodic forest cover assessment that
provides “valuable inputs for planning, policy formulation and evidence-based decision-making”. Since
19.53% in the early 1980s, India’s forest cover has increased to 21.71% in 2021.
o Adding to this a notional 2.91% tree cover estimated in 2021, the country’s total green cover now
stands at 24.62%, on paper.
• While the Forest Survey of India (FSI) started publishing its biennial State of Forest reports in 1987, it has
been mapping India’s forest cover since the early 1980s.
• According to the FAO, forests are defined as land spanning more than 0.5 hectares with trees
higher than 5 meters and a canopy cover of more than 10 percent, or trees able to reach these
thresholds in situ. It does not include landthat is predominantly under agricultural or urban land
use.
• India counts all plots of 1 hectare or above, with at least 10% tree canopy density, irrespective of
land use or ownership, within forest cover. This disregards the United Nation’s benchmark that
does not include areas predominantly under agricultural and urban land use in forests. Hence
statement 1 is not correct.
• All land areas with tree canopy density of 40% and above are considered dense forests and those between
10-40% are open forests.
o Since 2003, a new category — very dense forest — was assigned to land with 70% or more
canopy density. Hence statement 3 is correct.
• Since 2001, isolated or small patches of trees — less than 1 hectare and not counted as forest — are
assessed for determining a notional area under tree cover by putting together the crowns of individual
patches and trees.
• In India, land recorded as forest in revenue records or proclaimed as forest under a forest law is
described as Recorded Forest Area.
o These areas were recorded as forests at some point due to the presence of forests on the land.
o They are divided into Reserved, Protected and Unclassed forests, Recorded Forest Areas
account for 23.58% of India. Hence statement 2 is not correct.
• Over time, some of these Recorded Forest Areas lost forest cover due to encroachment, diversion, forest
fire etc. And tree cover improved in many places outside the Recorded Forest Areas due to agro-forestry,
orchards etc.
o In 2011, when the FSI furnished data on India’s forest cover inside and outside Recorded Forest
Areas, it came to light that nearly one-third of Recorded Forest Areas had no forest at all.
• India State of Forest Report (ISFR) 2021

25 www.visionias.in ©Vision IAS


o The Total Forest and Tree cover is 24.62% of the geographical area of the country.
o The Total Forest cover is 7,13,789 sq km which is 21.71% of the geographical area of the country.
The Tree cover is 2.91% of the geographical area of the country.

Q 43.A
• The leaf is a lateral, generally flattened structure borne on the stem. It develops at the node and bears a
bud in its axil. The axillary bud later develops into a branch. Leaves originate from shoot apical meristems
and are arranged in an acropetal order. They are the most important vegetative organs for photosynthesis.
• A typical leaf consists of three main parts: leaf base, petiole and lamina.
• Leaves are often modified to perform functions other than photosynthesis. They are converted into
tendrils for climbing as in peas or into spines for defence as in cacti.
• The fleshy leaves of onion and garlic store food.
• In some plants such as Australian acacia, the leaves are small and short-lived. The petioles in these plants
expand, become green and synthesise food. Leaves of certain insectivorous plants such as pitcher plant,
venus-fly trap are also modified leaves.
• Hence option (a) is the correct answer.

Q 44.C
• Recently, the reptile, the Jeypore Hill Gecko (Cyrtodactylus jeyporensis) endemic to India, has been
included in Appendix II of the Convention on International Trade in Endangered Species (CITES). The
species is rare and was first described by a British officer and amateur naturalist Colonel Richard Henry
Beddome in 1878 and rediscovered by a team of researchers led by Ishan Agarwal in 2010 and 2011, after
a gap of over 130 years. Hence Statement 1 is correct.
• The wild reptile species are found in the Eastern Ghats and are known to be present in four locations
including southern Odisha and northern Andhra Pradesh. Its occurrence is estimated in less than
5,000 square kilometers of fragmented geographical area. The species resides below rock boulders in
high forested hills at an altitude of 1,100-1,400 meters.
• The animals listed under Appendix II may not necessarily be threatened with extinction, but trade in such
species should be controlled to ensure it does not threaten their existence.
• The proposal to include the gecko in Appendix II was made by India at the recently-concluded 19th
Conference of Parties (COP19) to CITES in Panama City. The proposal was adopted by the members of
the Working Group.
• In 2019, it was assessed for the International Union for Conservation of Nature (IUCN)’s Red List of
Threatened Species and was listed as ‘endangered’. Hence statement 2 is correct.

Q 45.C
• CNG is obtained by compressing natural gas (CNG) under high pressure while LNG (Liquefied
Natural Gas) is produced by cooling NG to -162 degrees celsius in the Cryogenic stage through
Liquefaction.
o It is kept in liquid form to increase quantities of NG that can be stored in tanks. Hence
statement 1 is correct.
o This liquid gas is 600 times smaller than the original volume and is half the weight of water. The
compressed fossil fuel, which is constituted almost wholly of methane— a potent greenhouse gas —
can be transported around the world by ship. After arriving at its destination, the cargo is regasified in
a floating terminal and redistributed through pipelines.
o The high cost of liquefaction and producing LNG has limited its market.
• Climate impact: A lot of energy is required to extract natural gas from a reservoir, transport it from the
gas field to the LNG facility for processing, chill gas to such low temperatures, and hold it at that
temperature before it is warmed and regasified following a long sea or train journey. LNG emits about
twice as much greenhouse gas as ordinary natural gas. Processing LNG is so energy- and carbon-
intensive that it can create almost 10 times more carbon emissions than piped gas. Hence statement 2
is correct.
• Supply of Liquefied Natural Gas in India: At present, the government has accorded the highest priority
in domestic gas supply to City Gas Distribution (CGD) for distribution to the domestic household
(cooking purpose-PNG domestic) and transport segment (CNG). The industrial and commercial users of
the CGD network also consume market-determined price gas including LNG based on their respective
techno-commercial considerations.
• At present, there is no cross-country LNG Pipelines in the country.
26 www.visionias.in ©Vision IAS
• Petroleum & Natural Gas Regulatory Board (PNGRB), under the PNGRB Act, 2006, has notified
Technical and Safety Regulations for Natural Gas Pipelines in line with the various national/ international
standards, and the authorized pipeline entity is bound to comply with the standards/specifications as
provided under notified Regulations. The share of natural gas in the primary energy mix is envisaged to
increase to 15% by 2030 by boosting domestic production and procuring LNG.
• LNG imports are under Open General Licensing (OGL) category and the establishment of LNG
infrastructure, including LNG terminals, is also under 100% FDI (automatic route).

Q 46.D
• The Global Environment Facility (GEF) is a multilateral fund dedicated to confronting biodiversity
loss, climate change, pollution, and strains on land and ocean health. Its grants, blended financing, and
policy support help developing countries address their biggest environmental priorities and adhere to
international environmental conventions.
o The World Bank established the Global Environment Facility (GEF) in 1991 in collaboration with the
United Nations Development Programme (UNDP) and the United Nations Environment Programme
(UNEP) to provide funding to protect the global environment.
o At the Rio Earth Summit in 1992, the GEF was restructured and separated from the World Bank
system to become a permanent, separate institution. However, since 1994, the World Bank has
served as a Trustee of the GEF Trust Fund and has provided administrative services. It is
headquartered in Washington, DC, USA. India is a GEF donor as well as a recipient.
o The GEF connects 184 member governments with sustainability leaders across civil society,
Indigenous Peoples, and the private sector, and works closely with other environmental financiers for
efficiency and impact. Over the past three decades, the GEF has provided more than $22 billion in
grants and blended finance and mobilized $120 billion in co-financing for more than 5,000 national
and regional projects.
• The GEF provides funding to assist developing countries in meeting the objectives of international
environmental conventions. The GEF serves as a "financial mechanism" for five conventions:
1. Convention on Biological Diversity (CBD),
2. United Nations Framework Convention on Climate Change (UNFCCC),
3. Stockholm Convention on Persistent Organic Pollutants (POPs),
4. UN Convention to Combat Desertification (UNCCD),
5. Minamata Convention on Mercury.
• The conventions, for which the GEF serves as a financial mechanism, provide broad strategic guidance to
the two governing bodies of the GEF: the GEF Council and the GEF Assembly.
• Hence option (d) is the correct answer.

Q 47.B
• Recent Context: The Ministry of Finance has approved India's first sovereign green bonds (SGB)
framework. The Union Budget 2022-23 announced the issuance of SGBs.
• A green bond is a fixed-income instrument designed to support specific climate-related or
environmental projects. SGBs are issued by the Government.
• Key highlights of framework:
o A ‘green project’ classification is based on the principles like: Encourages energy efficiency, reduces
carbon & GHG emissions etc.
o Eligible projects under the framework include: Renewable energy, clean transportation, water
and waste management, green building etc. Hence options 1, 2 and 3 are correct.
o It excludes, nuclear power generation, landfill projects, direct waste incineration, hydropower
plants larger than 25 MW etc.
o Green Finance Working Committee constituted to validate key decisions on issuance of SGB. Hence
options 4 and 5 are not correct.
o Its proceeds will be deposited with Consolidated Fund of India.

Q 48.D
• There are 106 existing national parks in India covering an area of 44,372.42 km2, which is 1.35% of the
geographical area of the country.
• With Raimona and Dihing National parks being notified, Assam became the state with the second-
highest number of national parks after Madhya Pradesh which has 11 National Parks.

27 www.visionias.in ©Vision IAS


• While Maharashtra has 6 National parks, Tamil Nadu has 5 and Gujarat State has 4 National
Parks in all.
• Hence, the correct order in decreasing order is Madhya Pradesh-Assam-Maharashtra-Tamil Nadu-
Gujarat. Hence option (d) is the correct answer.

.
28 www.visionias.in ©Vision IAS
Q 49.C
• Of the 720 species of butterflies found in Sikkim, Blue Duke was selected as the state
butterfly securing 57 percent votes in a recent online poll conducted by the Forest Department.
o Krishna Peacock (Papilio Krishna) secured 43 percent of the votes in the online polls to secure the
second position.
• About Blue Duke:
o Blue Duke is a native butterfly species of Sikkim.
o It is also called Bassarona durga.
o It was first discovered in the State in 1858.
o It represents Sikkim with its two unique colors:
▪ Blue represents the sky.
▪ White depicting snow-clad mountains of the Himalayas.
o It is found at an altitude below 1,500 meters in the Himalayas and in various districts of West Sikkim.
o It falls under Schedule 2 of the Wildlife Protection Act, of 1972.
• Hence option (c) is the correct answer.

Q 50.C
• Lake effect snow is common across the Great Lakes region during the late fall and winter. Hence
statement 1 is correct.
• Lake Effect snow occurs when cold air, often originating from Canada, moves across the open waters of
the Great Lakes. As the cold air passes over the unfrozen and relatively warm waters of the Great Lakes,
warmth and moisture are transferred into the lowest portion of the atmosphere.
• The air rises, clouds form, and grow into a narrow band that produces 2 to 3 inches of snow per hour or
more.
• Wind direction is a key component in determining which areas will receive lake-effect snow.
• Heavy snow may be falling in one location, while the sun may be shining just a mile or two away in either
direction. The physical geography of the land and water is also important.
• Lake-effect snow is strongly influenced by the differences between the amount of heat and moisture
at the lake's surface and in the air a few thousand feet above it. Hence statement 2 is correct.
• A big contrast creates conditions that help to suck water up from the lake, and thus more snowfall. A
difference of 25 degrees Fahrenheit (14 Celsius) or more creates an environment that can fuel heavy
snow. This often happens in late fall, when the lake water is still warm from summer and the cold air
starts sweeping down from Canada. More moderate lake-effect snows occur every fall under less extreme
thermal contrasts.
• The wind’s path over the lakes is important too. The farther cold air travels over the lake's surface, the
more moisture is evaporated from the lake. A long “fetch” – the distance over water – often results in
more lake-effect snow than a shorter one.
• Once the snow reaches land, elevation contributes an additional effect. Land that slopes up from the lake
increases lift in the atmosphere, enhancing snowfall rates. This mechanism is termed the “orographic
effect.”
• The Tug Hill plateau, located between the lake and the Adirondacks in western New York, is well known
for its impressive snowfall totals.

Q 51.C
• A fuel cell uses the chemical energy of hydrogen or another fuel to cleanly and efficiently produce
electricity. If hydrogen is the fuel, electricity, water, and heat are the only products. Fuel cells are
unique in terms of the variety of their potential applications; they can provide power for systems as
large as a utility power station and as small as a laptop computer. Hence, statement 1 is correct.
29 www.visionias.in ©Vision IAS
• Fuel cells can be used in a wide range of applications, including transportation, material handling,
stationary, portable, and emergency backup power applications. Because fuel cells can be grid-
independent, they’re also an attractive option for critical load functions such as data centers,
telecommunications towers, hospitals, emergency response systems, and even military applications for
national defense. Fuel cells have several benefits over conventional combustion-based technologies
currently used in many power plants and passenger vehicles.
• Fuel cells can operate at higher efficiencies than combustion engines, and can convert the chemical
energy in the fuel to electrical energy with efficiencies of up to 60%. Fuel cells have lower emissions than
combustion engines. Hydrogen fuel cells emit only water, so there are no carbon dioxide emissions and
no air pollutants that create smog and cause health problems at the point of operation. Also, fuel cells are
quiet during operation as they have fewer moving parts.
• Fuel cells are similar to batteries in that they produce electricity without combustion or
emissions. Unlike batteries, fuel cells do not run down or need to recharge—as long as there’s a
constant source of fuel and oxygen. Compared to conventional gasoline vehicles, fuel cell vehicles can
even reduce carbon dioxide by up to half if the hydrogen is produced by natural gas and by 90%, if the
hydrogen is produced by renewable energy, such as wind and solar. There are also no pollutants emitted
from the tailpipe—just water! Hence, statement 2 is correct.
• Fuel cell cars are very similar to traditional gasoline-powered cars.
• Similar to today’s gasoline vehicles, fuel cell electric cars can have a driving range of more than 300
miles on one tank of hydrogen fuel. They can refuel in just a few minutes and the fueling experience is
almost identical to a gas station. Since the “engine” (i.e., fuel cell) has no moving parts, one never needs
to change the oil. But because a fuel cell is more than twice as efficient as an internal combustion
engine, a fuel cell car travels farther on that tank of hydrogen than a traditional car would on gasoline.
This means someone only needs about half the amount of hydrogen, with double the fuel economy.

Q 52.A
• In Raman spectroscopy, a beam of photons, usually with wavelengths in the visible region, from a pulsed
laser impinges on a surface. The photons are scattered by molecules within the sample and give up energy
corresponding to vibrational levels within the scattering molecule. The scattered photons are analyzed by
a spectrometer, yielding a spectrum showing the energy losses, which are characteristic of the molecule
with which the photon interacts.
• Advantages of Raman Spectroscopy
o many organic and inorganic materials are suitable for Raman analysis. These can be solids, liquids,
polymers, or vapours.
o no sample preparation is needed.
o not interfered with by water. Hence, statement 2 is correct.
o non-destructive.
o highly specific like a chemical fingerprint of a material.
o Raman spectra are acquired quickly within seconds.
o samples can be analyzed through glass or polymer packaging.
o laser light and Raman scattered light can be transmitted by optical fibers over long distances for
remote analysis. Hence, statement 3 is correct.
o in Raman spectroscopy, the region from 4000 cm-1 to 50 cm-1 can be covered by a single recording.
o Raman spectra can be collected from a very small volume (< 1 μm in diameter).
o inorganic materials are easily analyzable with Raman spectroscopy.
• Disadvantages of Raman Spectroscopy
o can not be used for metals or alloys. Hence, statement 1 is not correct.
o the Raman effect is very weak. The detection needs sensitive and highly optimized
instrumentation. Hence, statement 4 is not correct.
o fluorescence of impurities or of the sample itself can hide the Raman spectrum. Some compounds
fluoresce when irradiated by the laser beam.
o sample heating through intense laser radiation can destroy the sample or cover the Raman spectrum.

Q 53.B
• Recently, Delhi Court ordered a narco test in a murder case putting the spotlight on the use of specialized
examination as an advanced tool for interrogation. Also known as truth serum, the narco-analysis test has
been used for solving crucial cases in the past. The test involves the intravenous administration of a

30 www.visionias.in ©Vision IAS


drug (such as sodium pentothal, scopolamine, and sodium amytal) that causes the person undergoing it
to enter into various stages of anesthesia.
• The Narco test is also known as the narcoanalysis test. During the test, sodium pentothal is injected
into the person’s body. Also known as the ‘truth serum’, this injection transports the accused to a
hypnotic state. Hence option (b) is the correct answer.
o In this state, it is believed, the accused is incapable of lying. But in India, the Narco analysis report is
not admissible as primary evidence in the court.
o A polygraph test is another form of physiological examination, but in this test, no substances are
injected into the person’s body. Instead, instruments to measure blood pressure, pulse rate,
breathing, sweat glands, and blood flow is attached to the person. After this, he/she would have to
answer some questions.
• Sodium benzoate is a widely used preservative found in many foods and soft drinks.
• A numerical value is assigned to each response to calculate if the person is lying or telling the truth.
• Erythropoietin (EPO): It is a peptide hormone that is produced naturally by the human body. It is
generally used for doping in sports. EPO is released from the kidneys and acts on the bone marrow to
stimulate red blood cell production. By injecting EPO, athletes aim to increase the concentration of red
blood cells and consequently their aerobic capacity.
• Benzodiazepines are the most widely used group of sedative drugs. Sedatives encompass a wide variety
of drugs with different mechanisms of action that can induce depression of the central nervous system
(CNS). Barbiturates are nonselective CNS depressants that used to be the mainstay of treatment to sedate
patients or to induce and maintain sleep. In modern medicine, they have been largely replaced by
benzodiazepines, primarily because they can induce tolerance, physical dependence, and serious
withdrawal symptoms.

Q 54.A
• A pulse oximeter is a small clip-like device that uses light beams to estimate the oxygen saturation of
the blood. Hence statement 1 is correct.
• According to a WHO training manual on pulse oximetry, if the oxygen saturation is 93% or lower, the
patient needs to be treated quickly.
• A saturation of less than 90% is a clinical emergency. Hence statement 2 is not correct.
• The oximeter can estimate the amount of oxygen in the blood without having to draw a blood sample.
• It can be attached to the fingertip, earlobe or toes. It is a painless test, taking less than two minutes.
• The oximeter has a small electronic processor and a pair of LEDs – one emitting red light and
another infrared.
• The oximeter shines these two lights which pass through the finger and the device senses what comes
through the other side.
• Oxygenated haemoglobin and deoxygenated haemoglobin absorb red and infrared light differently.
• Oxygenated haemoglobin absorbs more infrared light and allows more red light to pass through
than deoxygenated haemoglobin.
• The amount of light that is transmitted is measured by the processor and the device displays the oxygen
saturation or the percentage of oxygenated haemoglobin in the blood.
• A normal blood oxygen saturation rate is often considered between 95 per cent and 100 per cent.

Q 55.C
• Types of Intellectual Property Rights
• IPRs are fundamental to encouraging investment in research as without some form of protection, investors
and inventors would not be able to benefit from their creative efforts. Owners of rights can prevent
unauthorized use of their IP, stop copying, control distribution, and retain, license, or sell their IP.
• A number of common IPRs are outlined below:
o A patent protects an invention. It gives the holder an exclusive right to prevent others from
selling, making, and using the patented invention for a certain period (typically 20 years from
the filing date). Hence, statement 4 is correct.
o Copyright protects the expression of literary or artistic work. Protection arises automatically
giving the holder the exclusive right to control reproduction or adaptation. Hence, statement 1
is correct.
o A trademark is a distinctive sign which is used to distinguish the products or services of one
business from others. Trademarks are often closely linked to brands. Hence, statement 3 is
correct.
31 www.visionias.in ©Vision IAS
o A Design protects the form of outward appearance or aesthetic style of an object. Does not
protect functionality or unseen (internal) design elements. Hence, statement 2 is correct.
o A Database right prevents the copying of substantial parts of a database. The protection is not over the
form of expression of information but of the information itself, but in many other aspects database
right is similar to copyright.
o A trade secret is a formula, practice, process, design, or compilation of information used by a business
to obtain an advantage over competitors. Trade secrets are by definition not disclosed to the world at
large.

Q 56.D
• Recently, global leaders confirmed USD 2.6 billion in funding toward the Global Polio Eradication
Initiative’s (GPEI) 2022-2026 Strategy to end polio at the World Health Summit in Berlin.
• About Polio:
o Polio, in full poliomyelitis, also called infantile paralysis, acute viral infectious disease of the nervous
system that usually begins with general symptoms such as fever, headache, nausea, fatigue, and
muscle pains.
o The polio virus is a picornavirus (family Picornaviridae), a member of a group known as
enteroviruses that inhabit the human digestive tract.
o Humans are the only known natural hosts of the poliovirus. The virus enters the body most often
by the so-called fecal-oral route—that is, from fecal matter taken into the mouth through
contaminated food or fingers. Hence, statement 2 is correct.
o It can also enter by ingestion of droplets expelled from the throat of an infected person.
o Polio mainly affects children under 5 years of age. However, anyone of any age who is unvaccinated
can contract the disease.
o There is no cure for polio, it can only be prevented. Hence, statement 3 is correct.
o Polio vaccine, given multiple times, can protect a child for life.
o There are two vaccines available: oral polio vaccine and inactivated polio vaccine. Both are effective
and safe, and both are used in different combinations worldwide, depending on local epidemiological
and programmatic circumstances, to ensure the best possible protection to populations can be
provided.
o India rolled out the Pulse Polio Immunization Programme on 2 October 1994, when the country
accounted for around 60% of the global polio cases. Within two decades, India received ‘Polio-free
certification’ from World Health Organization on 27 March 2014, with the last polio case being
reported in Howrah in West Bengal on 13 January 2011. Hence, statement 1 is correct.

Q 57.C
• Recently, scientists in the United Kingdom (UK) have successfully tested a new form of cancer
therapy, ‘Base Editing’ for the time in a patient with T-cell Acute Lymphoblastic Leukemia (T-
ALL). CAR-T therapy was ineffective in this case.
• CAR-T therapy involves adding a gene to T-cells that cause them to seek out and destroy cancerous cells.
The modified cells are known as CAR-T cells.
• First, an individual’s own T-cells are removed, which are then modified and reintroduced to the
individual.
• The problem with such an approach (besides the expense) is that very often when an individual is really
sick, it is simply impossible to obtain enough healthy T-cells to create CAR-T cells.
• While donors can provide healthy T-cells to an individual, these T-cells from a foreign body are going to
attack every single cell in that patient’s body, making the treatment counterproductive.
• Thus, scientists have resorted to what is known as base editing– through this technique of genetic
editing, they make it possible for one donor to supply T-cells to multiple recipients, without the
traditional risks associated with it. Hence statement 2 is correct.
• In less than a decade, the advent of CRISPR-Cas9 has made gene editing easier and faster than ever. But
CRISPR, as well as earlier genome editing tools, have a series of technical limitations, especially when it
comes to making very small modifications in the genome — of as little as one nucleotide base within a
DNA chain.
• Base editors have emerged as a technology that enables precise single base pair changes, and therefore has
a host of potential clinical utilities.
• Base editing is a novel technology that has the potential to generate gene knockouts or to correct certain
errors or mutations in the DNA of intact cells.
32 www.visionias.in ©Vision IAS
• Bases are the language of life. Just as letters in the alphabet spell out words that carry meaning, the
billions of bases in our Deoxyribonucleic Acid (DNA) spell out the instruction manual for our body. A
misarrangement in the sequence of bases may cause cancer.
• Using the technique of base editing, the molecular structure of just one base in a genetic code can be
altered, effectively changing its genetic instructions. Hence statement 1 is correct.
• Genetic code refers to the instructions contained in a gene that tells a cell how to make a specific protein.
• Most pathogenic mutations that cause human disease are single nucleotide polymorphisms that only
require a single nucleotide change to correct the mutation.

Q 58.C
• Total internal reflection (TIR) is the optical phenomenon in which waves arriving at the interface
(boundary) from one medium to another (e.g., from water to air) are not refracted into the second
("external") medium, but completely reflected back into the first ("internal") medium. It occurs when the
second medium has a higher wave speed (i.e., lower refractive index) than the first, and the waves are
incident at a sufficiently oblique angle on the interface. TIR occurs not only with electromagnetic waves
such as light and microwave but also with other types of waves, including sound and water waves.
o The refractive index of the material of the core is higher than that of the cladding. When a signal (
light ) is directed at one end of the fibre at a suitable angle, it undergoes repeated total internal
reflections along the length of the fibre and emerges at the other end. Hence option 1 is correct.
o A mirage is an optical illusion that deceives travellers moving in the desert during hot summer days.
In a hot desert, the layer of air closest to the ground is hot and therefore rare, being in direct contact
with the hot sands. Successive upper layers are cooler and denser. A ray of light moving down
towards the earth keeps bending away from the normal as it travels into lower hotter layers. At a large
enough angle of incidence (i<c)the ray will undergo total internal reflection. The reflected ray will
now keep bending towards the normal as it travels upwards into ever denser layers of air. This is why
light is observed to have a curved path as the refractive index keeps increasing as you move up from
the surface. Hence option 2 is correct.
o At the time of diamond polishing, specific cuts are made on the diamond surfaces to ensure light
rays strike the inner surface at angles greater than the critical angle as much as possible. Thus light
rays entering a diamond undergo multiple total internal reflections before coming out, thereby
giving it a sparkling effect. Hence option 3 is correct.
• Scattering of light: Scattering happens when light moves from one medium to another, such as air or a
glass of water, and a portion of the light is absorbed by the medium’s particles, followed by subsequent
radiation in a specific direction. The intensity of scattered light depends on the size of the particles and the
wavelength of the light. It occurs when it travels through a medium containing some obstacles suspended.
For example, when sunlight travels through the earth’s atmosphere before reaching the earth’s surface and
gets scattered by the obstacles like atoms, molecules, dust particles, water droplets, etc., present in the
atmosphere.
o The blue colour of the sky can be explained by the Rayleigh scattering of sunlight. As the
wavelength of blue colour is roughly half the wavelength of red colour, therefore, the intensity of
scattered blue light is about 2^4 times more than that of red light. Due to this, the blue colour
dominates, and the sky appears blue. Also, Blue is scattered more than other colours because it travels
as shorter, smaller waves. This is why we see a blue sky most of the time. Hence option 4 is not
correct.
o During sunrise and sunset, the sun is near the horizon. The rays from the sun must travel a larger
distance through the dense part of the atmosphere to reach the observer. During this course of the
journey of the light rays, most of the blue light gets scattered away. The red colour, which has the
highest wavelength among the colours in the visible spectrum, gets scattered the least. Due to this, the
light from the sun during sunrise and sunset entering the observer’s eye consists mainly of light of
higher wavelengths like red. Thus, the sun looks reddish at the time sunrise and sunset. Hence option
5 is not correct.

Q 59.A
• A nucleic acid containing deoxyribose is called deoxyribonucleic acid (DNA) while that which
contains ribose is called ribonucleic acid (RNA).
• Both DNA & RNA contain Adenine, Guanine & Cytosine.
• The fourth base is Thymine in DNA & Uracil in RNA.
• The structure of DNA is a double strand (helix) whereas RNA is a single-strand molecule.
33 www.visionias.in ©Vision IAS
• RNA mutation rate is higher than the DNA mutation rate. Hence statement 1 is correct.
• Hydrogen bonds bind the nitrogenous bases of the two separate polynucleotide strands to make double-
stranded DNA.
• DNA replication takes place in the nucleus while RNA replication takes place in the cytoplasm.
Hence statement 2 is correct.
• DNA viruses are stable while RNA viruses are unstable. Hence statement 3 is not correct.
• DNA occurs only in one form in any organism but there are 3 types of RNA present in an organism: –
mRNA, rRNA, and tRNA.

Q 60.D
• An image formed by a plane mirror is erect and of the same size as the object.
• If the reflecting surface curves inwards (concave), it is called a concave mirror.
• If the reflecting surface curves outwards (convex), then it is a convex mirror.
• The image formed by a concave mirror can be smaller or larger in size than the object. The image may
also be real or virtual.
• Concave mirrors are used for many purposes.
• You might have seen doctors using concave mirrors for examining eyes, ears, nose, and throat.
• Concave mirrors are used in optical instruments such as ophthalmoscopes. Ophthalmoscope consists
of a concave mirror with a hole in the center. The doctor focuses through the small hole from behind the
concave mirror while a light beam is directed into the pupil of the patient’s eye. This makes the retina
visible and makes it easy for doctors to check.
• Large concave mirrors are used to focus sunlight to produce heat in the solar furnace. They are also
used in solar ovens to collect a large amount of solar energy in the focus of the concave mirror for
heating, cooking, melting metals, etc.
• The reflectors of torches, headlights of cars, and scooters are concave in shape.
• The mirrors used as side mirrors in scooters are convex mirrors. Convex mirrors can form images
of objects spread over a large area. So, these help the drivers to see the traffic behind them.
• Hence, option (d) is the correct answer.

Q 61.C
• Dhrupad is the oldest vocal and instrumental style existing form of North Indian classical music.
o The origin of this music is linked to the recitation of Sama Veda, the sacred Sanskrit text.
o Though a highly developed classical art with a complex and elaborate grammar and aesthetics, it is
also primarily a form of worship, in which offerings are made to the divine through sound or
nada.
o Dhrupad can be seen at different levels as a meditation, a mantric recitation, a worship, a yoga or
tantra based on the knowledge of the nadis and chakras and also purely as a performing art
portraying a universe of human emotions.
o One significant characteristic of Dhrupad is the emphasis on maintaining purity of the ragas and
the swaras (notes). Hence statement 1 is correct.
o According to some accounts, Dhrupad was sung in temples, the singer facing the divinity. The
compositions were prayers addressed to Hindu gods. Hence statement 2 is correct.
o Later Dhrupad evolved into a highly sophisticated and complex musical form, still retaining its sacred
character.
o The language of composition also changed from Sanskrit to Brijbhasha, sometime between the
12th and the 16th centuries.
o Dhrupad music has two major parts (each of the two parts is further subdivided into several),
Alap and Dhrupad. Alap is sung without words, Dhrupad (The fixed composition part) is sung
with the accompaniment of a two-headed barrel drum called Pakhawaj.
o A major part of the compositions being sung nowadays were written in the 16th century and later.
o About six centuries ago, Dhrupad music came to be patronized by the royal courts and its
complex rendering became intended for highly sophisticated royal audiences. Among other
topics, several compositions were written in praise of emperors.
o Tansen was an Indian musician and poet who was an important figure in the North Indian
tradition of Hindustani classical music (Dhrupad). He was greatly esteemed for his dhrupad and
raga compositions and for his vocal performances.
▪ He joined the court of the Mughal emperor Akbar, who was well known for his patronage of
the arts. Hence statement 3 is correct.
34 www.visionias.in ©Vision IAS
▪ Tansen became one of the navaratnas - a collection of the 9 most talented intellectuals and
artists in the court.
▪ His skill was honoured with the title Mian (“Master”).
▪ Tansen was the title given to him by Raja Vikramjit of Gwalior.

Q 62.D
• Ellora is an archaeological site, 30 km away from Aurangabad city built by the Rashtrakuta rulers.
o Well known for its monumental caves, Ellora is a World Heritage Site.
o The 34 “caves”, actually structures excavated out of the vertical face of the Charanandri hills,
being Buddhist, Hindu, and Jain rock-cut temples and monasteries were built between the 5th
and 10th centuries.
o Ravana shaking Mount Kailasa is a sculpture panel that can be found at Dhumar Lena (Cave
29) at Ellora caves. Hence option (d) is the correct answer.
• Additional Information:
• Located in a tiny hamlet of Sittanavasal in the Pudukkottai district of Tamil Nadu, Sittanavasal
Cave is a Jain complex of caves dating back to the 2nd century. It is believed that it was the settlement
of certain monks from the eastern parts of India, who came here to preach lessons of Jainism and spread
the positives of ascetic life.
o Later on, this site became a major Jain center.
o Sittanavasal Cave is generally considered a rock-cut monastery and houses several murals made out
of vegetable and mineral dyes in numerous colours. These paintings are definitely masterpieces in
their own ways considering the period in which they were made.
• The Group of Monuments at Mammalapuram or Mahabalipuram is a collection of 7th- and 8th-
century CE religious monuments in Tamil Nadu.
o The monuments were built during the Pallava dynasty.
o The group contains several categories of monuments:
▪ Ratha temples: They are with monolithic temples carved in the shape of chariots.The best-
known are the five monolithic structures known as the Five Rathas or the Pandava Rathas:
Dharmaraja Ratha, Bhima Ratha, Arjuna Ratha, Nakula-Sahadeva Ratha and Draupadi
Rathas. Other ratha monuments at Mahabalipuram include the late-7th-century Ganesha Ratha.
▪ Some of the major temples are:
➢ Shore Temple: It consists of a large temple, two smaller temples and many minor shrines,
open halls, gateways, and other elements, much of which is buried by sand
➢ Olakkanesvara temple: It was built in the early 8th century from grey granite by King
Rajasimha. The walls of the temple depict the Ravananugraha legend from the Ramayana and
a relief of Dakshinamurti (Shiva as a yoga teacher).
➢ Mukundanayanar temple: It has ratha-like architecture.
▪ Rock reliefs: The best-known rock relief is the Descent of the Ganges also known as Arjuna's
Penance or Bhagiratha's Penance.
▪ Cave temples: Varaha cave; Kotikal cave; Dharmaraja cave; Ramanuja Cave.
• Elephanta Caves also known as Gharapuri caves are a UNESCO World Heritage Site and
a collection of cave temples predominantly dedicated to the Hindu god Shiva.
o They contain rock cut stone sculptures that show syncretism of Hindu and Buddhist ideas.
o Elephanta sculptures are known for their remarkable qualities of surface smoothness, elongation and
rhythmic movement.
▪ The image of Maheshmurti at Elephanta dates back to the early sixth century CE. It is
considered a masterpiece and the most important sculpture in the Elephanta caves. The central
head is the main Shiva figure whereas the other two visible heads are of Bhairava and Uma.

Q 63.C
• Recently, a massive Varaha statue (wild boar) dating 5th century AD was discovered in
Bandhavgarh Tiger Reserve, Madhya Pradesh.
• Wild boar in Indian mythology is seen as a symbol of valor and royalty, for example, Vishnu as the
wild boar is depicted carrying the earth goddess, Bhoodevi, on his snout.
• Various dynasties have presented wild boar in different forms ranging from sculptures to murals.
o The Western Chalukyas not only used the boar as their dynastic emblem, they even adopted a unique
imperial title Sri-Prithivi-Vallabha, which translates roughly to ‘The Fortunate Lord of the Earth’ or

35 www.visionias.in ©Vision IAS


to ‘Fortune’s Favourite and Earth’s Beloved’: as a form of Vishnu, Varaha is married both to the
goddess of fortune (Sri) and the goddess of earth (Prithvi).
o The royal emblem of Vijayanagara was Varaha / wild boar. Varaha is the avatar of the Hindu god
Vishnu, in the form of a boar.
o The Western Gangas minted coins with Kannada and Nagari legends, the most common feature
on their coins was the image of an elephant, which was also their royal emblem on the obverse and
floral petal symbols on the reverse, while, the royal emblem of Rashtrakutas was The Golden eagle.
• Hence, option (c) is the correct answer.

Q 64.D
• The Sangeet Ratnakara of Sarangdeva is one of the most important works on music. Hence pair 1 is
correctly matched.
o It is one of the longest works on music in Sanskrit. It is a very comprehensive exposition of the
subject, dealing with all aspects. Ever since it was written it has remained the standard work on music.
o Sangitaratnakara is divided into seven adhyayas dealing with : Svara, Prabandha, Nrtiya, Kaga, Tala,
Prakirnaka, Vadya.
• Chaturdandi Prakasika is a Sanskrit treatise written by the musicologist Venkatamakhin. Hence
pair 2 is correctly matched.
o It was a landmark in the chronicles of Carnatic music.
o The name means 'Exposition of the four channels through which a raga manifests itself.
o Chaturdandi Prakashika explicates the melakarta system of classifying ragas.
• Abhinaya Darpana is considered to be one of the earliest among the preserved Sanskrit books on
performing arts written by Nandikesvara. Hence pair 3 is correctly matched.
o It is the first treatise exclusively dealing with the technique of classical Indian dance.
o The Abhinaya Darpana, according to Nandikesvara is an abridgment of the Bharatarnava, an
exposition on the Art of dancing.

Q 65.B
• Recently, a specimen is now in the British Museum repository for which the info-document says:
o “Aware that visual elements on coins were fundamental to their acceptance and success as currency,
he (Mohammed bin Sam) issued coins which maintained a visual tradition begun in India 1,000
years previously, on the coinage of Kushan king Kanishka II.”
• India has a long tradition of coinage with images of gods and goddesses. According to historians, the
Kushans, who hailed from the Central Asian region and ruled till the 3rd century AD, were the first
to use the image of Goddess Lakshmi on their coins, along with Ardochsho, the Iranic Goddess of
wealth. The Kushans also depicted Oesho (Shiva), moon deity Miro and Buddha in their
coinage. Hence statement 1 is not correct.
• Mohammed bin Sam, known to us as Mohammed Ghori issued coins with Goddess Lakshmi after
defeating Prithviraj Chauhan in the Battle of Tarain in 1192 AD. Mohammed bin Sam was aware that
visual elements on coins were fundamental to their acceptance and success as currency. Hence, he issued
coins that maintained a visual tradition began in India 1,000 years ago. Hence statement 2 is correct.
• The Vijayanagara kings used coinage with Hindu idols. Harihara –II (1377-1404) introduced coins
that had Brahma-Saraswati, Vishnu-Lakshmi and Shiva-Parvati. Some of these coins can be seen in
the museum set up in Hampi by the Archaeological Survey of India (ASI) at Kamalapur. The
Vijayanagara coins continued in circulation even after the kingdom was snuffed out in 1565 and
commanded a premium when French traveller Tavernier visited the region. Hence statement 3 is not
correct.
• The British East India Co. at Madras Presidency minted coins labelled as the Three Swamy Pagoda, which
depicts Lord Balaji flanked by Sridevi and Bhudevi on either side. They were issued between 1691 and
1740.
• The tradition of minting coins to win the confidence of local people continued when the French and Dutch
minted coins showing Vishnu between 1715 and 1774 and Goddess Kali in the late 17th century.

Q 66.C
• Recent Context: At a distance of about 160 km from Jharkhand's capital Ranchi, Giridih's Parasnath
hill range has drawn national and international attention as Jain protests erupted across the
country against the central government notification promoting eco-tourism at the Jain pilgrimage
site.
36 www.visionias.in ©Vision IAS
o Surrendering to the Jain protest across the country, the Centre halted all tourist activities at the
Sammed Shikharji mountain peak on January 6, which is Marang Buru (supreme deity) for the
Santhal community.
o Adivasi communities and leaders have begun raising their demands to ascertain their right to their
native land.
• 'Sammed Shikharji' also known as Sammet Shikharji or simply Shikharji is a holy site for the Jain
community located at Parasnath Parvat, Giridih district in Jharkhand. It is considered to be one of the
biggest pilgrimage sites by both the Digambaras and the Svetambaras followers of Jainism. Hence,
statement 1 is not correct.
• It is believed that it is the place where 20 of the 24 Jain Tirthankaras along with many other
monks attained ‘moksha‘ or salvation after meditating. However, the first Jain Tirthankara
Rishabhanatha attained 'moksha' on Mount Kailas in the Himalayas. Hence, statement 2 is not
correct.
• According to the beliefs of the Jain community, Shikharji is ranked with Ashtapad, Girnar, Dilwara
Temples of Mount Abu and Shatrunjaya as the ‘Svetambara Pancha Tirth’ or the five principal
pilgrimage shrines. Hence, statement 3 is correct.
• The reference of the temple and the hill is found in the holy textbook Jnatradharmakatha, of
Jainism.

Q 67.A
• Faced with the rising demand for indigo in Europe, the Company in India looked for ways to expand the
area under indigo cultivation. Indigo was in great demand in Europe during the eighteenth and nineteenth
centuries. So the company looked for ways to expand Indigo cultivation. In 1788 only about 30 percent of
the indigo imported into Britain was from India. By 1810, the proportion had gone up to 95 percent.
• There were two main systems of indigo cultivation – nij and ryoti. Within the system of nij
cultivation, the planter produced indigo in lands that he directly controlled. He either bought the
land or rented it from other zamindars and produced indigo by directly employing hired laborers. Hence,
option (a) is the correct answer.
• The planters found it difficult to expand the area under nij cultivation. Till the late nineteenth century,
planters were, therefore, reluctant to expand the area under nij cultivation. Less than 25 percent of the land
producing indigo was under this system. The rest was under an alternative mode of cultivation – the ryoti
system.
• Under the ryoti system, the planters forced the ryots to sign a contract, an agreement (satta). At times
they pressurized the village headmen to sign the contract on behalf of the ryots. Those who signed the
contract got cash advances from the planters at low rates of interest to produce indigo. Under the ryoti
system, Indigo was cultivated mainly on lands owned by the peasants.

Q 68.B
• In 1923, Sarojini Naidu proposed the formation of a Hindustani Seva Dal at a Congress
convention. Jawahar Lal Nehru was appointed its first President and it later came to be known as
the Seva Dal. Hence option (b) is the correct answer.
• According to the resolution at Kakinada, the Dal was to work under the supervision of the Congress
party's working committee. Umabai Kundapur was the founding president of the women's wing of the
Dal.
• At the 1924 Belgaum Congress session, the Seva Dal was assigned the task of managing sanitation and
security for the first time.
• The organization played a significant role in India’s freedom struggle, training Congress workers and
instilling in them military discipline and the passion to fight for liberty through the ideals of truth and non-
violence. Several freedom fighters such as Rajguru, Rajendra Prasad and Subhash Chandra Bose
were members of Seva Dal.

Q 69.C
• Pattachitra painting: This form of art is closely related to the cult of Shri Jagannath and the temple
traditions in Puri. It is believed to have originated as early as the 12th century. Hence, statement 1 is
correct.
• The art is based in the eastern Indian states of Odisha, West Bengal, and parts of Bangladesh. People
in Odisha practice it to this day.

37 www.visionias.in ©Vision IAS


• Painting process:
o Traditionally the painters are known as chitrakars. Chitrakars follow a traditional process of preparing
the canvas.
o A gauze-like fine cotton cloth is coated with white stone powder and gum made out of tamarind
seeds. This makes the canvass ready to accept the paint, made of natural colors.
o The gum of the kaitha tree is the chief ingredient, used as a base for making different pigments
by adding available raw materials.
o Upon completion of a painting, the canvas is held over a charcoal fire and lacquer is applied to the
surface.
o In recent days, the Chitrakars have also painted on palm leaves and Tussar silk and have created wall
hangings and showpieces.
• Significant features:
o Pattachitra is predominantly icon painting. Hence, statement 2 is correct.
▪ In art, an icon mostly refers to a religious painting. It served as an object of devotion. Iconic
artworks are represented in a traditional style of wood and used in religious practices. Icons also
refer to portraits, statues, symbols or figures. However, not every representation of religious
identity is an icon.
o A floral border is a must around the paintings, and so is the use of natural colors. Hence,
statement 3 is correct.
o The paintings are executed primarily in profile with elongated eyes.
o Generally, there are no landscapes, perspectives, and distant views.
o The dress style has Mughal influences.
• Some of the popular themes of this religious art are:
o The Badhia (a depiction of the temple of Jagannath);
o Different "Vesas" of Shri Jagannath, Balabhadra, and Subhadra.
o Krishna Lila (an enactment of Jagannath as Lord Krishna displaying his powers as a child);
o Dasabatara Patti (the ten incarnations of Lord Vishnu); and
o Panchamukhi (a depiction of Lord Ganesh as a five-headed deity).
• It received a GI Tag in 2008.

Q 70.D
• Ministry of Education (MoE) has approved the ‘New India Literacy Programme’ for the period
FYs 2022-2027, to cover all the aspects of Adult Education to align with the National Education
Policy (NEP).
• The beneficiaries under the scheme are non-literates of the age of 15 years and above in all
states/UTs. Hence statement 1 is not correct.
• The scheme has five components, namely;
o Foundational Literacy and Numeracy, Critical Life Skills (which include, financial literacy, digital
literacy, legal literacy, healthcare and awareness, childcare and education, family welfare, etc.),
o Basic Education (includes preparatory (classes 3 - 5), middle (classes 6- 8), and secondary stage
(classes 9-12) equivalency),
o Vocational Skills (Skill development will be a part of the continuous learning process for neo-literates
to obtain local employment),
o Continuing Education (This includes engaging holistic adult education courses in arts, sciences,
technology, culture, sports, and recreation, as well as other topics of interest or use to local learners).
o Formation and involvement of SHGs, Voluntary & User Groups, and other community-based
organizations may be encouraged.
• Implementation:
o The scheme will be implemented via online mode using “Online Teaching, Learning and Assessment
System (OTLAS) through volunteerism.
o The students of the schools registered under Unified District Information System for Education
(UDISE) and around 50 lakh teachers of government, aided and private schools will participate
as volunteers. Hence statement 2 is not correct.
• Features:
o School will be the Unit for implementing the scheme.o Schools to be used for conducting surveys of
beneficiaries and Voluntary Teachers (VTs).
o Foundational Literacy and Numeracy will be imparted through Critical Life Skills to all non-literates
in the age group of 15 years and above.
38 www.visionias.in ©Vision IAS
o Performance Grading Index (PGI) for State/UT at the district level.
o CSR/Philanthropic Support may be received by hosting ICT support and providing volunteer support.
• Target:
o The target for Foundational Literacy and Numeracy for FYs 2022-27 is 5 crore learners at one crore
per year by using “Online Teaching, Learning and Assessment System (OTLAS)

Q 71.C
• Recently, a Supreme Court bench led by Justice M.R. Shah said that there was no justification to
deny a woman belonging to a Scheduled Tribe community the “right of survivorship” under the
Hindu Succession Act and directed the Centre to examine the provisions of the Hindu Succession
Act and, if necessary, amend the statute to extend it to the Scheduled Tribes.
• The Mitaksara is a legal commentary on the Yajnavalkya Smriti best known for its theory of
"inheritance by birth." It is codified as the Hindu Succession Act, of 1956 which governs succession and
inheritance of property. In 2020, the SC has expanded the Hindu women’s right to be the coparcener (joint
legal heir) and inherit ancestral property on terms equal to male heirs. Hence, statement 1 is correct.
• It applied to everyone who is not a Muslim, Christian, Parsi, or Jew by religion. Buddhists, Sikhs,
Jains, and followers of Arya Samaj, and Brahmo Samaj, are also considered Hindus for this law. In a
Hindu Undivided Family, several legal heirs through generations can exist jointly. Hence, statement 2 is
not correct.
• According to Section 2(2) of the Hindu Succession Act, the statute which guarantees equal shares for
male and female heirs is not applicable to members of Scheduled Tribe communities. Hence,
statement 3 is correct.

Q 72.B
• Bharat Ratna is the highest civilian award in the country. It is awarded in recognition of exceptional
service/performance of the highest order in any field of human endeavor. It is treated on a different
footing from Padma Award. The recommendations for Bharat Ratna are made by the Prime Minister to
the President of India. No formal recommendations for Bharat Ratna are necessary. The number of Bharat
Ratna Awards is restricted to a maximum of three in a particular year. The government has conferred
Bharat Ratna Award on 45 persons to date.
• Padma Awards, which were instituted in the year 1954, are announced every year on the occasion of
Republic Day except for brief interruption(s) during the years 1978 and 1979 and 1993 to 1997.
• The award is given in three categories, namely,
o Padma Vibhushan (second highest civilian award) for exceptional and distinguished service;
o Padma Bhushan (third highest civilian award) for distinguished service of a high order; and
o Padma Shri (fourth highest civilian award) for distinguished service.
• The few Indian personalities who have been awarded all four civilian awards are
o Satyajit Ray was an Indian filmmaker who worked prominently in Bengali cinema. Ray received
numerous honors, including India's highest civilian award, Bharat Ratna (1992). He has got all 4
civilian honors. Hence option 2 is correct.
o Ustad Bismillah Khan, often referred to by the honorific title Ustad, was an Indian musician credited
with popularizing the shehnai. He has got all 4 civilian honors.
o Bhupen Hazarika: He was an Indian playback singer, lyricist, musician, poet, actor, filmmaker, and
politician from Assam, widely known as Sudha Kontho. Hence option 4 is correct.
o Pandit Bhimsen Gururaj Joshi, also known by the honorific prefix Pandit, was one of the greatest
Indian vocalists from Karnataka, in the Hindustani classical tradition.
• Dr. APJ Abdul Kalam got his Padma Bhushan in 1981 and the Padma Vibhushan in 1990. In 1997, Kalam
received India's highest civilian honor, the Bharat Ratna. He was never awarded the Padma Shri. Hence
option 1 is not correct.
• Shri Atal Bihari Vajpayee received only the 2 most prestigious of civilian awards; Bharat Ratna (2015)
and Padma Vibhushan (1992). Hence option 3 is not correct

Q 73.A
• Before Punchhi Commission (2007), the Sarkaria Commission was set up in 1983 by the central
government of India. The Sarkaria Commission's charter was to examine the central-state relationship on
various portfolios and suggest changes within the framework of the Constitution of India.
• The Punchhi Commission was constituted by the Government of India in 2007 as a Commission on
Centre-State relations to relook into the problems and issues in Centre-State relations in India ever since
39 www.visionias.in ©Vision IAS
these issues were earlier looked into by the Sarkaria Commission which had submitted its report in 1988.
It was chaired by Justice Madan Mohan Punchhi who was formerly the Chief Justice of India. Hence
statement 1 is not correct.
• The Commission gave 312 recommendations in its report. One of the major recommendations was to
amend Articles 355 and 356 to protect the interests of the States by trying to curb their misuse by the
Centre. It said that the center should try to bring only the specific troubled area under its jurisdiction and
that too for a brief period, not more than three months. Hence statement 2 is correct.
• The other important recommendations of the Punchhi Commission are mentioned below:
o To facilitate the effective implementation of the laws on List III subjects, it is necessary that some
broad agreement is reached between the Union and states before introducing legislation in Parliament
on matters in the Concurrent List.
o The Union should be extremely restrained in asserting Parliamentary supremacy in matters assigned
to the states. Greater flexibility to states in relation to subjects in the State List and “transferred items”
in the Concurrent List is the key to better center-state relations.
o The period of six months prescribed in Article 201 for the State Legislature to act when the bill is
returned by the President can be made applicable for the President and also to decide on assenting or
withholding assent to a state bill reserved for the consideration of the President.
o While selecting Governors, the Central Government should adopt the following strict guidelines as
recommended in the Sarkaria Commission report and follow its mandate in letter and spirit:
▪ He should be eminent in some walk of life
▪ He should be a person from outside the state
▪ He should be a detached figure and not too intimately connected with the local politics of the state
▪ He should be a person who has not taken too great a par politics generally and particularly in the
recent past
o Governors should be given a fixed tenure of five years and their removal should not be at the sweet
will of the Government at the Centre.

Q 74.D
• The Supreme Court has agreed to examine a PIL challenging changes made to Article 19 in the first
amendment to the Constitution in 1951.
• The petitioner believes that the First Constitution (Amendment) Act violated the basic structure doctrine
of the Constitution of India, which affects the freedom of speech and freedom to trade.
• Another claim is that the amendment neglects national security by dropping the expression ‘tends to
overthrow the State’, which raises grave concern in the context of the dangers posed to the concept of the
secular democratic republic by radicalism, terrorism, and religious fundamentalism.
• About the First Amendment Act
o The first Amendment Act was enacted during the tenure of Pt. Jawaharlal Nehru.
o The first Amendment is called the Constitutional (First Amendment) Act, of 1951.
o The Amendment made the following important changes to the Constitution.
▪ Article 19 of Fundamental Rights was amended.
▪ Article 15 (4) - Article 15 of Fundamental Rights was amended to insert Article 15 (4) to allow
the State to take steps for the advancement of the weaker sections e.g. backward classes (socio-
economic; educationally backward classes), scheduled tribes, and scheduled castes.
▪ Ninth Schedule - Article 31A was inserted, creating the Ninth Schedule to the Constitution. Laws
contained in this Ninth Schedule would be shielded from judicial review to prevent the courts
from striking down laws intended to abolish the Zamindari system as unconstitutional.
▪ Citizenship as criteria - Article 376 was amended to make ‘Citizen of India’ as eligibility criteria
for appointment as Chief Justice of such High Court, or as Chief Justice or other Judge of any
other High Court.
• Hence option (d) is the correct answer.

Q 75.C
• Rule 18 of the Conduct of Election Rules, 1961 is related to persons eligible to vote by post (or postal
ballot). It says the following types of persons are entitled to vote by post (upon the fulfillment of certain
conditions):
o Special voters;
o Service voters;

40 www.visionias.in ©Vision IAS


o Voters on election duty;
o Electors subjected to preventive detention
• Special voters: A special voter is a voter defined in the Representation of People Act, of 1950, and can
be the President of India, the Vice-president, A Union Minister, or the Governor of a state. The special
voters can vote in person and also in case they are not available, they can vote by post. A special
voter who wishes to vote by post at an election will have to send an intimation to the returning officer
which has to reach the returning officer at least ten days (10 days) before the date of the poll, and once
the intimation is received of the returning officer will issue a postal ballot paper to him.
• Voters on election duty: This category includes different government officers and employees on poll
duty. A voter on election duty who wishes to vote by post at an election has to send an application to the
returning officer which should reach the returning officer within at least seven days (7 days or less than
seven days as the returning officer may allow) before the date of the poll; and if the returning officer is
satisfied that the applicant is a voter on election duty, he will issue a postal ballot paper to him. Voters on
election duty can also vote in person (thus not only through postal ballot) by taking the permission
of the returning officer and the procedure is defined under Rule 20 of the Conduction of Election
Rules, 1961. Hence option 4 is not correct.
• Electors under preventive detention: Any elector subjected to preventive detention, within fifteen days
of the calling of an election, has to send an intimation to the returning officer that he wishes to vote by
post, specifying his name, address, electoral roll number, and place of detention. Hence option 2 is
correct.
• Service voter: A service is a voter having service qualification. According to the provisions of subsection
(8) of Section 20 of the Representation of People Act, 1950, service qualification means
o Being a member of the armed Forces of the Union; Hence option 1 is correct.
o Being a member of a force to which provisions of the Army Act, 1950 (46 of 1950), have been made
applicable whether with or without modification ;
o Being a member of an Armed Police Force of a State, and serving outside that state; or
o Being a person who is employed under the Government of India, in a post outside India.
• The wife of a Service Voter, if she is ordinarily residing with him, shall also be eligible to be enrolled in
the part meant for Service Voters in the constituency concerned. Hence option 3 is correct.
• Members of the Armed Forces here refers to persons in 'active service' as defined in the Army Act
1950 i.e. or in similar situations in the navy and the air force when the persons are engaged in
operations against an enemy, or engaged in military operations in, or on the line of march to, a country or
place wholly or partly occupied by an enemy, thus rendering them unavailable to vote in person.

Q 76.D
• Article 30 of the Indian Constitution states the right of linguistic or religious minorities to establish
and administer educational institutions. It says: “All minorities, whether based on religion or language,
shall have the right to establish and administer educational institutions of their choice.”
• The National Commission of Minorities Act, declares six communities as minority communities. They are
- Muslims, Christians, Buddhists, Sikhs, Jains, and Zoroastrians (Parsis). The linguistic minority can be
described as those minorities residing in the territory of India, or any part thereof, which have a distinct
language or script of their own.
• Hence option (d) is the correct answer.

Q 77.C
• The Constitution of India has not stated or explicitly mentioned powers to change the capital of the state.
Legal experts argue that this power is implicit in Article 2 and Article 3 of the Constitution of
India. Hence, statement 1 is not correct.
• Presently, some states of India like Himachal Pradesh and Maharashtra, have multiple capitals.
Maharashtra has two capitals — Mumbai and Nagpur — with the latter being the winter capital of the
state. Himachal Pradesh has two capitals – Shimla and Dharamshala. Shimla is the summer capital of
Himachal Pradesh and Dharamshala is the winter capital of Himachal Pradesh. Hence, statement 2 is not
correct.
• There is no law in India that prohibits a state from having multiple capitals. Recently the Andhra
Pradesh Legislative Assembly passed a bill that give shape to the state government's plan of having three
capitals — the executive capital in Visakhapatnam, the legislative one in Amaravati, and the judicial in
Kurnool. Thus, it can be assumed that states in India can have separate capitals for legislative,
judicial, and administrative functions. Hence, statement 3 is correct.
41 www.visionias.in ©Vision IAS
Q 78.B
• Military bases are facilities directly possessed and operated by or for the military to shelter military
equipment and personnel and facilitates training and operations.
• These are established overseas to enable a nation to project power, thereby influencing events abroad. The
United States is the largest operator of military bases abroad- 38 named bases.
• Indian military bases located outside Indian territory:
o Tajikistan: Farkhor Air Base- It is operated by Indian Air Force in collaboration with the Tajik Air
Force. It is India's first military base overseas. Chabahar Port in Iran facilitates Indian transport access
to Farkhor Air Base. Hence pair 2 is correctly matched.
o Bhutan: Indian Military Training Team (IMTRAT)- It is a training mission of the Indian Army in
Bhutan to train the personnel of the Royal Bhutan Army (RBA) and the Royal Bodyguard of Bhutan
(RBG).
o Oman: India has established a listening post at Ras al Hadd and berthing rights for the Indian Navy
at the Muscat naval base. There is also an establishment at Duqm for the IAF and Indian
Navy. Hence pair 1 is correctly matched.
o Mauritius: In 2015, India signed an agreement with Mauritius for the development of Agalega
Island. The Agalega Island is in Mauritius and not in Seychelles. Hence pair 4 is not correctly
matched.
▪ While there have been speculative reports in the media about an Indian military base in Agalega,
the Government of Mauritius has categorically denied such reports and reiterated that there is no
agreement between Mauritius and India for the creation of a military base in Agalega.
o Seychelles: The Indian government supported the construction of a system of six coastal surveillance
radars in Mahe, Alphonse, Farquhar, Astove, and Assumption Islands which are linked to the
Indian surveillance system. The Assumption Island is in Seychelles and not in Mauritius. Hence
pair 3 is not correctly matched.
▪ India’s plans for a military base in Seychelles was first announced during a trip by India’s prime
minister to the islands in 2015. Though both countries signed a 20-year agreement in 2018 to
build an airstrip and a jetty for its navy on Assumption Island, the agreement has not been passed
by the Seychelles parliament.

Q 79.D
• Today, several industrial applications depend on hydrogen, but most are obtained by techniques, such as
steam reforming or electrolysis, not entirely free from the involvement of fossil fuels.
• Some living organisms, such as microalgae and bacteria, are the basis of processes capable of
producing hydrogen in a completely eco-sustainable way.
• Microalgal hydrogen production is made possible by biological processes directly or indirectly,
depending on sunlight, or by fermentation processes and thermochemical technologies for biomass
conversion.
42 www.visionias.in ©Vision IAS
• Photofermentation refers to fermentation processes that employ sunlight as an energy source for
photosynthesis. Instead of using sugar, this pathway utilizes light as a source of energy.
• The dark fermentation pathway is light-independent. It performs heterotrophic fermentation using
microbes or microalgae. Green algae rich in carbohydrates are the most common type of microalgae used
to produce biohydrogen in this process.
• Indirect biophotolysis is performed by microalgae under anoxic conditions. Under these conditions,
microalgae can produce biohydrogen through fermentation or respiration.
• Microalgae biomass is frequently utilized to generate various types of bioenergy (Ge et al., 2020), with
biohydrogen being one of the most promising categories of bioenergy that can be generated from
microalgae.
• Biohydrogen can be generated from microalgae using two alternative processes: i) direct production of
biohydrogen by microalgae, and ii) microalgal biomass utilization as a fermentation substrate by other
microbes.
• Hence, option (d) is the correct answer.

Q 80.A
• The carbon stock refers to the amount of carbon stored in forests in the form of biomass, soil,
deadwood, and litter. Hence statement 1 is correct. More the carbon stock, the higher would be the
forest's capacity to absorb and sequester carbon dioxide (CO2), the main greenhouse gas in the
atmosphere, through photosynthesis. A robust forest ecosystem is a vital sink of carbon.
• Forest Survey of India (FSI) assessed the carbon stock in India’s forests for the first time in 2004
and then biennially since 2011. Hence statement 2 is correct.
• Among the Indian States, Arunachal Pradesh has the maximum carbon stock in forests (1023.84
million tonnes), followed by Madhya Pradesh (609.25 million tonnes). The per-hectare forest carbon
stock among different States/UTs indicates that Jammu & Kashmir is contributing the maximum per-
hectare carbon stock of 173.41 tonnes. Hence statement 3 is not correct.
• The carbon stock for 2021 has been estimated at 7,204.0 million tonnes.
o There is an increase of 79,4 million tonnes of carbon stock as compared to the estimates of the 2019
report.
o State-wise Maximum carbon stock: Arunachal Pradesh > Madhya Pradesh > Chhattisgarh >
Maharashtra.State-wise Maximum per hectare carbon stock: Jammu & Kashmir > Himachal
Pradesh > Sikkim > Andaman & Nicobar Island.
• Soil organic carbon is the largest pool of forest carbon followed by Above Ground Biomass (AGB),
Below Ground Biomass (BGB), Litter, and dead wood. As compared to the 2019 assessment, maximum
changes have been observed in AGB and dead wood.

Q 81.A
• The ICAR-National Bureau of Animal Genetic Resources (NBAGR) is a research institute under the
Indian Council of Agricultural Research (ICAR), which is responsible for maintaining a national
database of animal genetic resources in India. NBAGR is located in Karnal, Haryana, and was
established in 1984.
43 www.visionias.in ©Vision IAS
• The functions and details of ICAR-NBAGR are as follows:
o Identification and characterization of animal genetic resources: NBAGR is responsible for
identifying and characterizing the animal genetic resources in India. This includes registering new
varieties of Indigenous breeds of animals. This involves collecting information on the various
breeds of animals, including their physical characteristics, performance, and genetic makeup.
o In the last year, the ICAR-National Bureau of Animal Genetic Resources (NBAGR) has
registered 10 new breeds of livestock species, including cattle, buffalo, goats, and pigs. This has
taken the total number of indigenous breeds to 212 as of January 4, 2023. Hence option (a) is the
correct answer.
o The 10 new breeds included three new cattle breeds (Kathani, Sanchori, Masilum), one buffalo
breed (Purnathadi), three goat breeds (Sojat, Karauli, Gujari), and three pig breeds (Banda,
Manipuri Black, Wak Chambil).
• Other Functions:
o Conservation of animal genetic resources: NBAGR is involved in the conservation of animal
genetic resources through various methods such as cryopreservation, in-vitro conservation, and live
animal conservation. The aim is to ensure that the genetic diversity of the animals is preserved for
future generations.
o Utilization of animal genetic resources: NBAGR is also involved in the utilization of animal genetic
resources by promoting their use in breeding programs and research.
o Certification of animal breeds: NBAGR provides certification for new breeds of livestock and
poultry, which ensures their genetic purity and authenticity. The certification process involves
verifying the breed's pedigree and assessing its genetic characteristics.
o Development of new technologies: NBAGR conducts research on animal genetics and breeding to
develop new technologies for improving livestock productivity and genetic diversity.
o National database of animal genetic resources: NBAGR maintains a national database of animal
genetic resources, which serves as a repository of information on animal breeds in India. This
database is accessible to researchers, breeders, and other stakeholders for research and development
purposes.

Q 82.D
• International Treaty on Plant Genetic Resources for Food and Agriculture (ITPGRFA): It is a
legally binding comprehensive agreement adopted in November 2001 at Rome during the 31st session
of the Food and Agriculture Organization of the United Nations, which entered into force on June 29,
2004, and currently has 149 Contracting Parties, including India. Hence statement 1 is correct.
• The treaty provides solutions to achieve food and nutritional security as well as climate resilient
agriculture. Countries are interdependent for PGRFA and consequently, a global order is essential to
facilitate access and benefit sharing.
• It formally acknowledges the enormous contribution of indigenous people and small-holder farmers as
traditional custodians of the world’s food crops.
• The treaty was aimed at:
o recognizing the significant contribution of farmers to the diversity of crops that feed the world;
o establishing a global system to provide farmers, plant breeders, and scientists with access to plant
genetic materials; and
o ensuring that recipients share the benefits they derive from the use of these genetic materials with the
countries where they have originated.
• It also provided the international legal framework needed for the establishment of the Svalbard Global
Seed Vault in Norway. Hence statement 3 is correct.
o It is also known as Seed Treaty as it is a comprehensive international agreement for ensuring food
security through the conservation, exchange, and sustainable use of the world's Plant Genetic
Resources for Food and Agriculture (PGRFA).
o India is a signatory to the treaty. Hence statement 2 is correct.
• NOTE: Protection of Plant Varieties and Farmers’ Rights (PPV&FR) Act, 2001
▪ It aims to protect Farmers’ and breeders’ rights.
▪ According to the act, a farmer is entitled to save, use, sow, resow, exchange, share, or sell his
farm produce including seed of a variety protected under the PPV&FR Act, 2001 except the brand
name.
▪ The Act is compliant with Article-9 of the Seed Treaty.

44 www.visionias.in ©Vision IAS


Q 83.B
• The United Nations Environment Assembly was created in June 2012, when world leaders called for
UN Environment to be strengthened and upgraded during the United Nations Conference on
Sustainable Development, also referred to as RIO+20.The establishment of the Environment Assembly
was the culmination of decades of international efforts, initiated at the UN Conference on the Human
Environment in Stockholm in 1972 and aimed at creating a coherent system of international
environmental governance. Hence statement 1 is not correct.
• The United Nations Environment Assembly is the world’s highest-level decision-making body on the
environment, with a universal membership of all 193 Member States. It addresses the critical
environmental challenges facing the world today. Understanding these challenges and preserving and
rehabilitating our environment is at the heart of the 2030 Agenda for Sustainable Development. Hence
statement 3 is correct.
• The Environment Assembly meets biennially to set priorities for global environmental policies and
develop international environmental law. Through its resolutions and calls to action, the Assembly
provides leadership and catalyses intergovernmental action on the environment. Hence statement 2 is
correct.
• The 5th UN Environment Assembly, 2022 in Nairobi concluded with 14 resolutions to strengthen
actions for nature to achieve the Sustainable Development Goals. The first and crucial resolution in
which the world’s ministers for the environment agreed to establish an Intergovernmental
Negotiating Committee with the mandate to forge an international legally binding agreement to end
plastic pollution.

Q 84.B
• The International Union for Conservation of Nature (IUCN) Red List of Threatened Species is the
global standard for assessing the risk of extinction that individual species of animal, fungus, and plant
faces. But we also need an optimistic vision of species conservation that presents a road map for
recovery.
• To achieve this, the Red List assessment process has been expanded to include new classifiers of species
recovery and conservation impact, known as the Green Status of Species. Hence option (b) is the
correct answer.
• The IUCN Green Status of Species complements the Red List by providing a tool for assessing the
recovery of species’ populations and measuring their conservation success. In 2020, Green Status of
Species assessments became an optional part of Red List assessments.
• It is misconception, frequently encountered in consultation, that a species assessed on a Green
“List” is no longer in need of conservation.
• How Does the Green Status of Species Define Recovery?
o A species is fully recovered if it is present in all parts of its range, even those that are no longer
occupied but were occupied prior to major human impacts/disruption; AND
o It is viable (i.e., not threatened with extinction) in all parts of the range; AND
o It is performing its ecological functions in all parts of the range.
• These factors contribute towards a “Green Score” ranging from 0–100%, which shows how far a
species is from its "fully recovered" state.

Q 85.A
• Recently, 18th Meeting of the Persistent Organic Pollutants Review Committee to the Stockholm
Convention (POPRC-18) was held.
o The committee concluded its review of four of the five chemicals under consideration.
▪ It recommended listing of Dechlorane Plus (flame retardant) and UV-328 (stabiliser) under
Annex A of the Stockholm Convention.
▪ On medium chain chlorinated paraffins (flame retardant) and Long-Chain
Perfluorocarboxylic Acids (PFCAs), risk management evaluations will be prepared for
consideration at the next Committee meeting. Hence option (a) is the correct answer.
➢ Flame retardants are chemicals that are applied to materials to prevent the start or slow
the growth of fire.
➢ They have been used in many consumer and industrial products since the 1970s, to decrease
the ability of materials to ignite.
o On chlorpyrifos (a pesticide), committee decided to defer its consideration of the draft risk profile.

45 www.visionias.in ©Vision IAS


• About Stockholm Convention:
o The Stockholm Convention is a global treaty to protect human health and the environment from
persistent organic pollutants (POPs).
▪ POPs are chemicals that remain intact in the environment for long periods, become widely
distributed geographically, accumulate in the fatty tissue of living organisms and are toxic
to humans and wildlife.
▪ POPs circulate globally and can cause damage wherever they travel.
o In implementing the Convention, Governments will take measures to eliminate or reduce the release
of POPs into the environment. It is legally binding.
o India ratified the Stockholm Convention in 2006.
o Ministry of Environment, Forests and Climate Change had notified the 'Regulation of POP
Rules' in 2018, under the Environment (Protection) Act, 1986.

Q 86.D
• Recently, at the 15th Conference of Parties (COP15) to the UN Convention on Biological Diversity
“Kunming-Montreal Global Biodiversity Framework” (GBF) was adopted.
o GBF includes 4 goals and 23 targets for achievement by 2030.
o The U.N. biodiversity conference concluded in Canada's Montreal.
o The first part of COP 15 took place in Kunming, China and reinforced the commitment to address the
biodiversity crisis and the Kunming-Declaration was adopted by over 100 countries.
• Key Targets of the GBF:
o Effective conservation and management of at least 30% of the world’s lands, inland waters, coastal
areas and oceans, with emphasis on areas of particular importance for biodiversity and ecosystem
functioning and services.
o Have restoration completed or underway on at least 30% of degraded terrestrial, inland waters,
and coastal and marine ecosystems. Hence option 1 is correct.
o Stop the extinction of known species, and by 2050 reduce tenfold the extinction risk and rate of all
species (including unknown).
o Reduce to near zero the loss of areas of high biodiversity importance,including ecosystems of high
ecological integrity.
o Cut global food waste in half and significantly reduce over consumption and waste generation.
o Reduce by half both excess nutrients and the overall risk posed by pesticides and highly hazardous
chemicals.
o Reduce risk from pesticides by at least 50% by 2030. Hence option 3 is correct.
o Reduce nutrients lost to the environment by at least 50% by 2030. Hence option 2 is correct.
o Reduce pollution risks and negative impacts of pollution from all sources by 2030 to levels that are
not harmful to biodiversity and ecosystem functions.
o Sustainably manage areas under agriculture, aquaculture, fisheries, and forestry and substantially
increase agroecology and other biodiversity-friendly practices.
o Tackle climate change through nature-based solutions.
o Reduce the rate of introduction and establishment of invasive alien species by at least 50% by
2030. Hence option 4 is correct.
o Secure the safe, legal and sustainable use and trade of wild species by 2030. Hence option 5 is
correct.
o Green up urban spaces.

Q 87.B
• The Central Highlands:
o They are bounded to the west by the Aravali range. The Satpura range is formed by a series of
scarped plateaus on the south, generally at an elevation varying between 600-900 m above the mean
sea level. This forms the northernmost boundary of the Deccan plateau. It is a classic example of the
relict mountains which are highly denuded and form discontinuous ranges. Hence option (a) is not
correct.
o The extension of the Peninsular plateau can be seen as far as Jaisalmer in the West, where it has been
covered by the longitudinal sand ridges and crescent-shaped sand dunes called barchans.
o The general elevation of the Central Highlands ranges between 700-1,000 m above the mean sea
level and it slopes towards the north and northeastern directions.

46 www.visionias.in ©Vision IAS


o An eastern extension of the Central Highland is formed by the Rajmahal hills, to the south of
which lies a large reserve of mineral resources in the Chotanagpur plateau.
• The Indian Desert:
o To the northwest of the Aravali hills lies the Great Indian desert. It is a land of undulating topography
dotted with longitudinal dunes and barchans.
o This region receives low rainfall below 150 mm per year; hence, it has an arid climate with low
vegetation cover. It is because of these characteristic features that this is also known as Marusthali. It
is believed that during the Mesozoic era, this region was under the sea.
o The underlying rock structure of the desert is an extension of the Peninsular plateau, yet, due to
extreme arid conditions, its surface features have been carved by physical weathering and wind
actions.
o Some of the well-pronounced desert land features present here are mushroom rocks, shifting dunes
and oases (mostly in its southern part).
o On the basis of the orientation, the desert can be divided into two parts: the northern part is
sloping towards Sindh and the southern towards the Rann of Kachchh. Hence option (b) is the
correct answer.
o Most of the rivers in this region are ephemeral. The Luni river flowing in the southern part of the
desert is of some significance.
• The Peninsular Plateau:
o Rising from the height of 150 m above the river plains up to an elevation of 600-900 m is the irregular
triangle known as the Peninsular plateau.
o Delhi ridge in the northwest, (extension of Aravalis), the Rajmahal hills in the east, Gir range in the
west and the Cardamom hills in the south constitute the outer extent of the Peninsular plateau.
However, an extension of this is also seen in the northeast, in the form of the Shillong and Karbi-
Anglong plateaus.
o The Peninsular India is made up of a series of patland plateaus such as the Hazaribagh plateau, the
Palamu plateau, the Ranchi plateau, the Malwa plateau, the Coimbatore plateau and the Karnataka
plateau, etc.
o This is one of the oldest and the most stable landmass of India. The general elevation of the plateau is
from the west to the east, which is also proved by the pattern of the flow of rivers.
o This Peninsular plateau has undergone recurrent phases of upliftment and submergence accompanied
by crustal faulting and fractures. The Bhima fault (northwestern part of the Peninsular
plateau) needs special mention, because of its recurrent seismic activities). These spatial variations
have brought in elements of diversity in the relief of the Peninsular plateau. The northwestern part of
the plateau has a complex relief of ravines and gorges. The ravines of Chambal, Bhind and Morena
are some well-known examples. Hence option (c) is not correct.
• The Arunachal Himalayas:
o These extend from the east of the Bhutan Himalayas up to the Diphu pass in the east. The general
direction of the mountain range is from southwest to northeast. Hence option (d) is not correct.
o Some of the important mountain peaks of the region are Kangtu and Namcha Barwa. These ranges are
dissected by fast-flowing rivers from the north to the south, forming deep gorges. The Brahmaputra
flows through a deep gorge after crossing Namcha Barwa.
o Some of the important rivers are the Kameng, the Subansiri, the Dihang, the Dibang and the Lohit.
These are perennial with a high rate of fall, thus, having the highest hydroelectric power potential in
the country.
o An important aspect of the Arunachal Himalayas is the numerous ethnic tribal community inhabiting
in these areas. Some of the prominent ones from west to east are the Monpa, Abor, Mishmi, Nyishi,
and Nagas. Most of these communities practice Jhumming. It is also known as shifting or slash-and-
burn cultivation.

Q 88.C
• North East India comprises of seven states commonly known as the “Seven Sisters” and the state of
Sikkim. The 'seven sisters' are Arunachal Pradesh, Assam, Manipur, Meghalaya, Mizoram, Nagaland and
Tripura.
• India has four of the total biodiversity hotspots in the world. They are the Himalayas, the Western
Ghats, the Indo-Burma region and the Sundaland. The part of India that falls in the Sundaland Hotspot
is the Nicobar Islands. Hence, option (b) is not correct.

47 www.visionias.in ©Vision IAS


• In the wild, lion-tailed macaques are only native to India. There, they live in the Western Ghats
hills and mountains of southwestern India, where they live in tropical rainforests and on elevated
mountainsides. Hence, option (a) is not correct.
• Northeastern states host a total of four Ramsar sites in India. They are Deepor Beel (Assam),
Loktak (Manipur), Rudrasagar lake (Tripura) and Pala wetland of Mizoram. Hence, option (c) is
the correct answer.
• Nathu La, one of the highest motorable roads in the world, is a mountain pass in the Himalayan
peaks that co-joins Sikkim and China. Situated on the Indo-Tibetan border 14450 ft. above sea level,
Nathu La is one of the most important Himalayan passes in the country. Bumla Pass, situated at an
altitude of over 15,000 ft in the Tawang region of Arunachal Pradesh is snowbound for almost the
entire year due to its steep elevation and presents a beautiful view of the Tibetan plateau. The Shipki La
is a mountain pass at the boundary of India and China in the western Himalayan region. It lies
between Tibet and Kinnaur district in Himachal Pradesh. Hence, option (d) is not correct.

Q 89.C
• Lonar Lake, also known as Lonar crater, is a notified National Geo-heritage Monument, located at
Lonar in Buldhana district, Maharashtra. Lonar Lake is an astrobleme created by
a meteorite impact during the Pleistocene Epoch. It is one of only four known hyper-velocity impact
craters in basaltic rock anywhere on Earth. Hence pair 1 is correctly matched.
• Wular Lake, in Kashmir, is one of the largest fresh water lakes in South Asia. It is located near
Bandipora town in Bandipora district of Jammu and Kashmir, India. The lake basin was formed as
a result of tectonic activity and is fed by the Jhelum River. Hence pair 2 is correctly matched.
• The Panchmuli lake, also known as ‘Dyke-3’ of the Sardar Sarovar Dam, was
developed (manmade) for tourists visiting the Statue of Unity (the 182-metre tall statue of Sardar
Vallabhbhai Patel) in Kevadia, Gujarat. It is located near the Narmada river basin. The Panchmuli Lake
features calm waters flanked by the jagged Vindhya mountains range, offering a picturesque and serene
experience to all visitors. Hence pair 3 is correctly matched.
• Kodaikanal Lake, also known as Kodai Lake, is a manmade lake located in the Kodaikanal city in Tamil
Nadu. Sir Vere Henry Levinge, the then Collector of Madurai, was instrumental in creating the lake in
1863, amidst the Kodaikanal town. The lake is star-shaped, centrally located in the town of Kodaikanal
and is surrounded by lush green hills of the northwestern Palani Hills range, which is the main watershed
for the lake. Hence pair 4 is not correctly matched.

Q 90.B
• India has engaged with its different trade partners comprehensively and signed many kinds of economic
agreements such as the Comprehensive Economic Partnership Agreements, Comprehensive Economic
Cooperation Agreements, Comprehensive Economic Cooperation and Partnership Agreements, etc. The
list of trade agreements signed by India is as under:

48 www.visionias.in ©Vision IAS


• In addition, India has signed the following 6 limited coverage Preferential Trade Agreements (PTAs):

• There is no trade agreement between the United States and India.


• The key difference between a Free Trade Agreement (FTA) and a Preferential Trade Agreement (PTA) is
that in a PTA there is a positive list of products on which duty is to be reduced; in an FTA, there is a
49 www.visionias.in ©Vision IAS
negative list on which duty is not reduced or eliminated. FTA is more comprehensive than PTA. India has
negotiated FTA with many countries e.g. Sri Lanka and various trading blocs as well e.g. ASEAN.
• Also, CEPA and CECA are not the same and have different characteristics.
o CECA is mainly concerned with tariff reductions and the elimination of all items that are considered
to be listed in tariff rate quota items.
o On the other hand, CEPA covers negotiation on the trade in services and investment, and other areas
of economic partnership. It may even consider negotiation on areas such as trade facilitation and
customs cooperation, competition, and IPR. In looking at the big picture, CEPA is much broader and
more complicated compared to CECA.
• Hence, option (b) is the correct answer.

Q 91.D
• Dumping is said to occur when goods are exported by a country to another country at a price lower than
its normal value. This is an unfair trade practice that can have a distortive effect on international trade.
• Anti-dumping duty is a measure to rectify the situation arising out of the dumping of goods and its
trade distortive effect. Thus, the purpose of anti-dumping duty is to rectify the trade distortive effect of
dumping and re-establish fair trade. The duty is aimed at ensuring fair trading practices and creating
a level-playing field for domestic producers vis-a-vis foreign producers and exporters. Hence
statement 1 is correct.
• The use of anti-dumping measures as an instrument of fair competition is permitted by the WTO. In
fact, anti-dumping is an instrument for ensuring fair trade and is not a measure of protection per se for the
domestic industry. It provides relief to the domestic industry against the injury caused by dumping. Hence
statement 2 is correct.
• According to global trade norms, a country is allowed to impose tariffs on such dumped products to
provide a level-playing field to domestic manufacturers. The duty is imposed only after a thorough
investigation by a quasi-judicial body, such as the Directorate General of Trade Remedies
(DGTR). The anti-dumping duties were imposed by the Central Board of Indirect Taxes and Customs
(CBIC), following recommendations of the commerce ministry's investigation arm Directorate General of
Trade Remedies (DGTR). Hence statement 3 is correct.

Q 92.B
• Social Progress Index (SPI) for states and districts made by the Institute for Competitiveness and
Social Progress Imperative was submitted to Economic Advisory Council- Prime Minister and was
released recently. Hence statement 2 is not correct.
• SPI is a comprehensive tool that can serve as a holistic measure of a country's social progress at the
national and sub-national levels. The index assesses states and districts based on 12 components
across three critical dimensions of social progress - Basic Human Needs, Foundations of Wellbeing,
and Opportunity. The index uses an extensive framework comprising 89 indicators at the state level and
49 at the district level. Hence statement 1 is correct.
o Basic Human Needs assess the performance of states and districts in terms of Nutrition and Basic
Medical Care, Water and Sanitation, Personal Safety and Shelter.
o Foundations of Wellbeing evaluates the progress made by the country across the components of
Access to Basic Knowledge, Access to Information and Communication, Health and Wellness, and
Environmental Quality.
o Opportunity focuses on Personal Rights, Personal Freedom and Choice, Inclusiveness, and Access to
Advanced Education.
• Based on the SPI scores, states and districts have been ranked under six tiers of social progress. The tiers
are Tier 1: Very High Social Progress; Tier 2: High Social Progress; Tier 3: Upper Middle Social
Progress; Tier 4: Lower Middle Social Progress; Tier 5: Low Social Progress; and Tier 6: Very Low
Social Progress.
• Puducherry has the highest SPI score of 65.99 in the country, attributable to its remarkable performance
across components like Personal Freedom and Choice, Shelter, and Water and Sanitation. Lakshadweep
and Goa closely follow it with scores of 65.89 and 65.53, respectively. Jharkhand and Bihar scored the
lowest, 43.95 and 44.47, respectively.

Q 93.A
• To facilitate the MSMEs to become internationally competitive, the Ministry of Micro, Small and
Medium Enterprises (MSME) and its organizations, through its various Schemes and Programmes, is
50 www.visionias.in ©Vision IAS
providing support to the Indian MSME sector, promoting bilateral cooperation with other countries;
giving them exposure to the international markets and foreign technology; sharing of experiences and best
management practices in the international arena, etc. One such programme of the Ministry is the
International Cooperation (IC) Scheme.
• The Scheme, being implemented by the Ministry of MSME, is an ongoing scheme of the Ninth Plan
(under implementation since 1996). Hence, statement 1 is correct.
• The main objectives of the Scheme are as below:
o Technology infusion and/or upgradation of Indian micro, small and medium enterprises (MSMEs).
o Promotion of the exports of MSMEs
o Modernisation of MSMEs
• The following organizations are eligible to apply under the scheme:
o State/Central Government Organisations. Hence, statement 3 is not correct.
o Industry/Enterprise Associations; and
o Registered Societies/Trusts and Organisations associated with the MSME.
• Eligibility Conditions: The eligibility conditions for financial assistance under the Scheme are:
o The organisation should be suitably registered (i.e., companies under the Companies Act, societies
under the Societies Act, etc.) with the primary objective of promotion and development of MSME.
o The organisation must be engaged in such activities for at least the last 3 years and have a good
track record. Hence, statement 2 is not correct.
o The organisation should have regular audited accounts for the past 3 years.
o Events, for which financial support under the Scheme is sought, must have significant international
participation.
• Assistance would be provided under the scheme to the eligible applicant organizations for participation in
international exhibitions/ Trade Fairs/ Buyer-Seller Meets held in foreign countries in order to showcase
Indian technologies, expose Indian MSMEs to the latest foreign technologies, access international buyers
and sellers and forge business alliances etc.

Q 94.A
• A capital account keeps a record of all the transactions related to assets between India and other countries.
This includes all kinds of investment assets like shares, debt, and property, or even corporate assets.
• Currently, there are limitations to how much capital can flow in and out of the country. So, India's capital
account is only partially convertible. When there are no restrictions, our capital account would be fully
convertible.
• Currently, an individual or high net-worth investor wanting to invest outside India can invest within
an overall limit of $250,000 per financial year under the Liberalised Remittance Scheme for any
permitted current or capital account transaction or a combination of both.
• There are a number of benefits of Capital Account Convertibility:
o A free capital account would make the rupee fully convertible. This can reduce the cost of
transactions and allow them to be conducted at a faster pace. Ease of access is always good for
the industry.
o It will allow global capital to move freely. This allows developing countries to access more money for
investments. This in turn helps improve savings and investments within the country, allows
domestic companies to borrow at cheaper rates, and thus accelerate growth.
o Moreover, the threat of FII sell-off can often force governments to take the right decisions like
narrowing fiscal deficit (the amount government borrows from the market).
• Yet, there are many risks of full convertibility.
o Many argue that capital flows are not always productive. For example, if foreigners are investing in a
country only for tax savings, then the flows may not lead to increased productivity and growth.
o Complete capital account convertibility can promote capital inflows into the country, but there is a
significant risk of capital outflows from the home country if conditions deteriorate. Any deterioration
in fiscal conditions, inflation management, the balance of payments, or any other macroeconomic
shock may cause a cessation or reversal of capital flows. This can lead to increased exchange rate
volatility and perhaps a crisis, similar to what occurred during the East Asian crisis.
• Hence option (a) is the correct answer.

Q 95.C
• Nominal variables are valued at their current market price and real variables are nominal variables
adjusted for inflation or deflation; valued at some base year. The base year or constant year should be
51 www.visionias.in ©Vision IAS
carefully selected. A change in nominal variable reflects the combined effects of changes in quantities and
changes in prices whereas a real variable provides a correct picture of a variable change or change in
quantity.
• Nominal GDP: It is also termed as monetary national income. It is defined as the value of goods and
services at current-year prices. It is obtained by multiplying goods and services produced in a current year
with the current year prices. It is a poor indicator of measuring economic growth.
• Real GDP: It is also termed as GDP at constant prices. It measures the actual growth of the economy. It is
obtained by multiplying goods and services produced in a current year with the base year prices. An
increase in real GDP means over time indicates an improvement in the performance of the economy. It
reflects changes in quantities.
• For example, let us consider an economy producing only rice and the base year (2014-15) price of rice is
rupees 5 per kg. In FY 2021, let the price of rice is 10 rupees/kg and the economy produced 100 kg of
rice.
o Then the Nominal GDP of the economy in FY 2021 is 1000 rupees (current production x current
price).
o But the real GDP in FY 2021 is 500 rupees (current production X base year price)
▪ In FY 2022, due to various factors let the production of rice declined to 50 kg and the price of rice
increased to 25 rupees per kg. In this situation,
o The nominal GDP in FY 2022 is 1250 rupees (current production X current prices).
o The real GDP is 250 rupees (current production X base year price)
▪ In the above situation, the nominal GDP has increased from 1000 rupees in FY 2021 to 1250
rupees in FY 2022, even though the production in the economy has declined. Hence economic
production in a country does not necessarily increase with the increase in nominal GDP. But
an increase in Real GDP necessarily implies an increase in the economic production of the
economy. Hence option 2 is correct.
• GDP Growth Rate is also known as the Economic Growth Rate, and it measures the change in the GDP of
the country in comparison to an earlier period. The amount of change is measured in percentage (%),
which serves as a determinant of economic health in the country and the possible growth in the future.
The GDP of a certain period, when set against another, can show a comparison that can be measured using
the given formula:
o Economic Growth = (GDP 2 - GDP 1) / GDP 1
• Nominal GDP growth is driven both by changes in real economic activity and by changes in prices.
From the above example, Nominal GDP growth in FY2022 is {(1250-1000)/1000} x 100 is equal to
25%. Hence even when there is positive growth in nominal GDP, it may not necessarily lead to an
increase in economic production. Hence option 1 is not correct.
o When the country has a positive growth rate of nominal GDP but negative growth rate of real
GDP, it indicates that the growth rate of inflation in the economy is much greater than the growth rate
of nominal GDP. In a way it indicates that, even though the GDP has increased in absolute terms,
much of the growth rate can be accounted to the increased prices of goods and services in the
economy.
• The Nominal Per Capita Income of a geographical location (say, a country, state, city, or others) measures
the amount of money earned by every person in that area. A country's Per Capita Income is determined by
dividing its national income by population. It determines the average income of a person in a country at
the current year's prices. This helps us evaluate the standard of livelihood and the quality of life of people
in the geographical location. Real Per Capita income is calculated by adjusting the inflation nominal one.
It is calculated by dividing real income or Real GDP by the population of a country.
o Real Percapita Income = Real National Income / Population
• Therefore, from the above equation, for a developing country like India whose population is
increasing, an increase in real per capita income may necessarily imply an increase in real National
Income. An increase in national income implies an increase in the flow of goods and services in the
economy provided that, national income is estimated at constant prices (i.e in real terms). Hence
option 3 is correct.

Q 96.B
• Recently, US officials applied the foreign direct product rule (FDPR) rule to China’s advanced
computing and supercomputer industry to stop it from obtaining advanced computing chips. The move
would ban any semiconductor manufacturing firm that uses American tools – which most do – from
selling advanced chips to China.
52 www.visionias.in ©Vision IAS
• The provision called the foreign direct product rule, or FDPR, was first introduced in 1959 to
control the trading of US technologies. It enables US regulators to extend their technology export
control powers far beyond America’s borders to transactions between foreign countries.
• It essentially says that if a product was made using American technology, the US government has the
power to stop it from being sold – including products made in a foreign country. Hence, option (b) is the
correct answer.
• Rules of origin: Rules of origin are the criteria needed to determine the national source of a product.
Rules of origin are important because many trade rules, regulations, and laws, provide differential
treatment (known as preferential rules of origin) to goods and products based on where they originate
from. The WTO Agreement on Rules of Origin arising out of the Uruguay round of WTO negotiations
was an attempt to harmonize the rules of origin for different countries.
• Most-favoured-nation (MFN): Under the WTO agreements, countries cannot normally discriminate
between their trading partners. Grant someone a special favour (such as a lower customs duty rate for one
of their products) and you have to do the same for all other WTO members. This principle is known as the
most-favoured-nation (MFN) treatment.
• The Technical Barriers to Trade (TBT) Agreement aims to ensure that technical regulations, standards,
and conformity assessment procedures are non-discriminatory and do not create unnecessary obstacles to
trade. At the same time, it recognises WTO members' right to implement measures to achieve legitimate
policy objectives, such as the protection of human health and safety, or protection of the environment. The
TBT Agreement strongly encourages members to base their measures on international standards as a
means to facilitate trade. Through its transparency provisions, it also aims to create a predictable trading
environment

Q 97.B
• What is Critical Information Infrastructure? The Information Technology Act of 2000 defines Critical
Information Infrastructure as a computer resource, the incapacitation or destruction of which shall
have debilitating impact on national security, economy, public health or safety. Hence statement 1
is correct.
• The government, under the IT Act of 2000, has the power to declare any data, database, IT network
or communications infrastructure as CII to protect that digital asset. Any person who secures
access or attempts to secure access to a protected system in violation of the law can be punished
with a jail term of up to 10 years. Hence statement 2 is not correct.
• Why is CII Classification and Protection Necessary?Global Practice: World over governments have been
moving with alacrity to protect their critical information infrastructure.
• Backbone of Countless Critical Operations: IT resources form the backbone of countless critical
operations in a country’s infrastructure, and given their interconnectedness, disruptions can have a
cascading effect across sectors.
• IT Failure leads to Crippling other Sectors: An information technology failure at a power grid can lead to
prolonged outages crippling other sectors like healthcare, banking services etc.
• Case of India:
• In October, 2020 as India battled the pandemic, the electric grid supply to Mumbai suddenly snapped
hitting the mega city’s hospitals, trains and businesses.Later, a study by a US firm claimed that this power
outage could have been a cyber-attack, allegedly from a China-linked group, aimed at critical
infrastructure.
• How are CIIs protected in India?
• NCIIPC as Nodal Agency:Created in January 2014, the National Critical Information
Infrastructure Protection Centre (NCIIPC) is the nodal agency for taking all measures to protect
the nation’s critical information infrastructure. Hence statement 3 is not correct.
• Mandate of NCIIPC:-It is mandated to guard CIIs from unauthorized access, modification, use, disclosure,
disruption, incapacitation or distraction.It will monitor and forecast national-level threats to CII for policy
guidance, expertise sharing and situational awareness for early warning or alerts.In the event of any threat
to critical information infrastructure the NCIIPC may call for information and give directions to the
critical sectors or persons serving or having a critical impact on Critical Information Infrastructure.The
basic responsibility for protecting the CII system shall lie with the agency running that CII.

53 www.visionias.in ©Vision IAS


Q 98.A
• Recently, markets regulator Securities and Exchange Board of India has proposed the concept of
blue bonds as a mode of sustainable finance, saying such securities can be utilised for various blue
economy-related activities, including oceanic resource mining and sustainable fishing.
• The blue bond is a debt instrument issued by governments, development banks or others to raise
capital from impact investors to finance marine and ocean-based projects that have positive
environmental, economic and climate benefits. Hence statement 1 is correct.
• SEBI has proposed the concept of blue bonds as a mode of sustainable finance. Currently, in India
there is no framework on issuance of Blue bonds.
• The Republic of Seychelles, in 2018, was the first to come with a sovereign blue bond. The objective
of the bond was to save Seychelles’ 115 islands, which are flanked by coral. Seychelles’ issue got the
support of the World Bank, as well as its Global Environment Facility. Hence statement 2 is not
correct.
• A blue bond is a relatively new form of debt instrument that is issued to support investments in healthy
oceans and blue economies. Blue Economy is sustainable use of ocean resources for economic growth,
improved livelihoods, and jobs.

• Blue bonds offer an opportunity for private sector capital to be mobilized to support the blue economy.
India can deploy blue bonds in various aspects of blue economy like oceanic resource mining, sustainable
fishing, national offshore wind energy policy etc.
• In India, the green debt securities can be issued by companies in unlisted and listed forms. While for
unlisted green debt securities, there are no specific guidelines over and above the general requirements for
issuance of debt securities, issuances of listed green debt securities are required to be in compliance with
the additional stipulations under the following regulations:
o The SEBI (Issue and Listing of Non-Convertible Securities) Regulations, 2021 (NCS Regulations);
o The SEBI (Listing Obligations and Disclosure Requirements) Regulations, 2015 (LODR
Regulations); and
o Chapter IX of the SEBI Operational Circular for Issue and Listing of Non-convertible Securities
(SEBI Operational Circular).
• These are collectively referred to as "Green Debt Securities Regulations".

Q 99.C
• Recently, the centre and the states have been in conflict regarding tax and duties imposed on petrol
and diesel. Though the centre has reduced excise duty on petrol and diesel, some states have not reduced
VAT imposed on fuel.
Petrol/Diesel Pricing in India:
• Public sector Oil Marketing Companies (OMCs) revise the retail prices of petrol and diesel in India
on a daily basis, according to the changes in the price of crude oil at global level. Price charged to dealers
includes the base price set by OMCs and the freight price. Hence statement 1 is not correct.

54 www.visionias.in ©Vision IAS


o The prices of petrol and diesel are not determined by the actual costs incurred by refiners on crude oil
sourcing, refining and marketing and allowing for profits. Rather a formula - Trade Parity Price
(TPP) - is the starting point for pricing these products. The TPP is determined based on prices of
these products prevailing in the international market assuming that 80% of the petrol and diesel is
imported and 20% is exported. To this is added the cost of inland freight, marketing costs and margins
charged by the oil companies, the dealer commission and finally the plethora of taxes levied by the
Central and State government.
o Retail price of petrol and diesel are made up of mainly 3 components - Base price (reflecting cost of
international oil), Central excise duty and State tax. States apply an ad valorem VAT or sales tax on
the base price, freight charges, excise duty and dealer commission on petrol and diesel. In fact,
Central and state taxes form a major chunk of the price of petrol and diesel in India.
o Central government taxes the production of petroleum products (Excise Duty) while states tax
their sale (Sales Tax/ Value Added Tax (VAT)). While excise duty rates are uniform across the
country, states levy sales tax/ VAT which varies across states. Hence statement 2 is correct.
o Excise duty consists of two broad components: tax component (i.e., basic excise duty), and cess
and surcharge component.
o Basic Excise Duty (BED), Special Additional Excise duty (SAED), Road & Infrastructure Cess
(RIC) and Agriculture & Infrastructure Development Cess (AIDC) together constitute Excise Duty on
petrol and diesel. Basic Excise Duty is shareable with states while SAED, RIC & AIDC are non-
shareable. Hence statement 3 is correct.
• Unlike excise duty, sales tax is an ad valorem tax, i.e., it does not have a fixed value, and is charged as a
percentage of the price of the product. The value of the excise duty component of the price structure is
fixed by centre. The value of the sales tax component is dependent on the other three components, i.e., the
price charged to dealers, dealer commission, and excise duty.

Q 100.A
• The Ministry of New and Renewable Energy (MNRE) launched the Atal Jyoti Yojana (AJAY) to
illuminate dark regions through the establishment of solar street lights.
o It is a sub-scheme under the Off-Grid and Decentralized Solar Application Scheme of the
Ministry of New and Renewable Energy.
o Phase I was implemented from September 2016- March 2018. Phase II is being implemented during
2018-19 and 2019-20.
o Energy Efficiency Services Limited (EESL) is implementing Atal Jyoti Yojana (AJAY). Hence,
statement 3 is correct.
• Under AJAY, solar LED Lights are being installed in rural, semi-urban, and urban areas which do
not enjoy adequate coverage of power. Hence, statement 1 is correct.
• Energy Efficiency Services Limited was founded in 2009.
o It is promoted by the Ministry of Power, Government of India as a Joint Venture of four reputed
public-sector undertakings NTPC Limited, Power Finance Corporation Limited, REC Limited, and
POWERGRID Corporation of India Limited.
o It focuses on solution-driven innovation without taking support of any subsidy from the Government.
• Under AJAY phase-I and II, EESL has installed over 1.97 lakhs of Solar LED streetlights in rural areas
of Uttar Pradesh, Assam, Bihar, Odisha, Jharkhand, Madhya Pradesh, Uttarakhand, Rajasthan,
and Gujarat. Hence, statement 2 is not correct.

55 www.visionias.in ©Vision IAS

You might also like